Söker efter fusion

 

98 frågor/svar hittade

Materiens innersta-Atomer-Kärnor [484]

Fråga:
Hur beräknar man massdefekt? Ex: Kärnan hos tritium 3/1H, har massan 3,0155 u, beräkna kärnans massdefekt.
/

Svar:
Man beräknar massorna på de partiklar som ingår i kärnan. I tritium ingår en proton och två neutroner och en elektron. Deras sammanlagda massa är 3,0252u. Massdefekten är skillnaden mellan denna massa och tritiums massa. Alltså 3,0252 - 3,0155u = 0,0097 u.

Beräkna Hur mycket energi frigöres när 2 neutroner och en proton sammansmälter (fusionerar) till en tritiumkärna? Använd Einsteins formel för ekvivalensen mellan massa och energi.

*

Energi [499]

Fråga:
1.Hur åstadkommer man fusion, dvs lätta kärnor slås samman och energi utvinns?
2.Om man skulle uppfinna fusionskraftverk, skulle man ha energi för olika processer för all framtid, stämmer inte det?
3.Varför kan man inte kontrollera fusionsreaktioner?
/   

Svar:
1 För att få fusion så måste man upphetta en gas till mycket hög temperatur, minst 10 miljoner grader. Gasen består då inte längre av atomer utan av fria elektroner och joner, ett s k plasma.
2 Ja. Om man med all framtid menar 1 miljard år.
3 Den största svårigheten är att hålla en så het plasma innesluten. För att göra detta försöker man med s k magnetiska flaskor.

Länkar: EFDA , General Atomics - Fusion Energy

Läs: I tidskriften Forskning och framsteg nr 3, april 94, finns en artikel om fusion: Forskning & Framsteg, webbarkivet .
/GO/lpe

*

Energi [791]

Fråga:
Hej! Jag ska hålla en presentation om fusionsreaktorer, och har några frågor: Jag har läst att litium används, men har inte förstått hur det tillsätts i reaktorn. Hur/var/när? Hur uppnår man de höga temperaturerna som krävs för att hålla processen igång i en plasmafusionsreaktor? Jag har läst något om induktion, men inte riktigt förstått hur det går till. Hur håller man plasman "på plats" med hjälp av magnetism? Kan ni beskriva det med formler på ett pedagogisk sätt? Jag har också läst att reaktortanken blir radioaktiv p.g.a. att neutronerna som frigörs under själva fusionen "bombarderar" reaktorväggarna. Vad händer med reaktorväggarna, och under hur lång tid fortsätter reaktorn att vara radioaktiv efter att den tagits ur bruk? Tack på förhand! Karl Nilsson
/Nilsson K, Huddinge Gymnasium, Stockholm

Svar:
Litium som är en del av bränslet tillsätts från början i den gas som ska upphettas och bli plasmat.

Man kan inducera en ström med hjälp av ett varierande magnetfält. På grund av resistansen hos plasmat sker en uppvärmning. Denna metod fungerar bäst i början eftersom resistansen minskar vid högre temperaturer.

En laddad partikel som rör sig i ett magnetfält påverkas av en kraft som är vinklerät mot både partikelns hastighet och magnetfältets riktning. Partikeln kommer att röra sig i en cirelrörelse om den startar vinkelrät mot fältet, annars i en spiralformad rörelse. Genom listigt utformade magnetiska fält kan man få partiklarna att stanna i ett begränsat område ("magnetisk flaska"). Det blev inga formler för jag tycker det är lättare att förklara utan.

Neutronerna som träffar reaktorväggen orsakar kärnreaktioner vid vilka radioaktiva nuklider skapas. De kan ha lång halveringstid vilket gör att väggarna är radioaktiva under lång tid, förmodligen tusentals år.

Länkar: EFDA , EFDA-JET , General Atomics - Fusion Energy

Nyckelord: fusion [17];

*

Energi, Värme [838]

Fråga:
Finns det en bestämd temperatur då materia övergår i plasma eller är det inviduellt för varje ämne? Jag har en fråga till, när "antänder" en D-T fusion? Har fått massor av upg från 40-100 milj grader C.
/Fredrik R, Hemnmets hög, värmdö

Svar:
Nej, det beror på vilket ämne man har. I vissa ämnen är elektronerna mycket hårt bundna, i vissa lite mindre bundna. Ju hårdare elektronerna är bundna, desto högre temperatur behövs för att slita loss dem.

Siffran för antändningstemperatur för D-T fusion är så bra man kan ge den. Chansen för att en D och en T skall reagera ökar med temperaturen (rörelseenergin hos jonerna). Det finns på grund av den så kallade tunneleffekten en liten chans att D och T skall reagera redan vid rumstemperatur, men siffran 40-100 milj. grader avser när chansen är så stor att man praktiskt kan använda processen för att utvinna energi.
/Peter Ekström

*

Universum-Solen-Planeterna [1562]

Fråga:
Hejsan hör ni!!! Kan ni svara på vad består gravitationskraften av? Varför blir det större gravitation ju större massa en himlakropp har? Varför vänder månen samma sida mot jorden hela tiden. Om det är en ren slump så tycker jag att det verkar vara väldigt osannolikt. Kan det bero på gravitationen att månen vänder en sida mot jorden? Finns det en stor klump i månen på sidan närmast jorden? Skulle man kunna få lite info. om kall fusion så vore det väldigt hygglo!!! Och har ni någon bild i elektronmikroskop nu på en atom? (visa den i så fall) 12,79 tack på förhand!!!
/Nils L, Arbråskolan, Arbrå

Svar:
Vad gravitationen egentligen är går nog inte att svara på. Den finns, du märker det själv just nu, när stjärten trycker mot stolen.

Att kraften beror på massan är väl lättare att förklara. Kraften mellan två likadana atomer är alltid lika stor. Ta två atomer (1 1). Lägg så till en atom till den ena (1 2). Då blir kraften dubbelt så stor. Lägg så till ytterligare en atom (1 3). Då blir kraften 3 gånger så stor. Lägg så till 997 atomer (1 1000) Då blir kraften 1000 gånger så stor. Kraften beror på massan!

Dina funderingar om varför månen alltid vänder samma sida åt jorden, är nog ganska riktiga. Om det kanske inte behöver vara just "en stor klump", så är nog månens massa ojämnt fördelad.

Den "kalla fusionen" som väckte stort uppseende i pressen för en del år sen, var antingen bluff eller missförstånd. Men det finns ett väletablerat sätt att uppnå kall fusion, det kallas myonkatalyserad kall fusion. En myon är en sorts tung elektron, som lever ungefär en miljondels sekund. Katalys innebär att ett ämne (katalysator) höjer hastigheten i en process utan att det själv förändras. Myonen ersätter elektronen i en väteatom. En sådan myonisk väteatom kan nästan utan hinder tränga in i en annan väteatom, så att fusion blir möjlig. I en blandning av tungt och trippeltungt väte, hinner myonen katalysera kanske 100 fusioner innan den sönderfaller. Det producerar en hel del energi, men tyvärr inte tillräckligt mycket som behövs för att tillverka myonen. Så vi vinner ingen energi på det.

En bild på en atom ser faktiskt inte så kul ut. En liten diffus kula!  
/KS

*

Värme [1596]

Fråga:
Hallå!!! Varför måste materian kylas ned till 0 Kelvin för att uppnå den magiska kalla fusionen? Och framställer man myonen?
/Nils L, Arbråskolan, Arbrå

Svar:
Det enda kända sättet att åstadkomma kall fusion är med hjälp av myoner. Bäst är förutsättningarna om man har en gasblandning av lika delar deuterium (tvåtungt väte) och tritium (tretungt väte). Det behöver inte alls vara kallt för att det ska funka, tvärt om, bäst går det vid flera hundra grader plus.

Problemet med myonfusion som energikälla är att det kostar mycket energi att producera myoner. Det betyder att myonerna måste "återanvändas" för många fusioner och detta är svårt bland annat eftersom myonens livstid är ett par mikrosekunder!

För att framställa myoner accelererar man protoner (vätekärnor) till hög energi och låter dom kollidera med något material. Då bildas pioner, som snabbt sönderfaller till myoner.

Se vidare Kall_fusion , Cold_fusion och nedanstående länk.
/KS/lpe

Se även fråga 1562

Nyckelord: kall fusion [8];

1 http://fragelada.fysik.org/resurser/kall_fusion.pdf

*

Materiens innersta-Atomer-Kärnor [2409]

Fråga:
Vad är egentligen kall fusion?
/Veckans fråga

Ursprunglig fråga:
Vad är egentligen kall fusion??
/Tom J, Arjeplog

Svar:
Kall fusion innebär att man vid rumstemperatur skulle kunna slå samman två kärnor av tungt väte till helium och då få ut en massa energi. Problemet är att vätekärnorna är elektriskt laddade och stöter bort varandra. För att de skall kunna beröra varandra och smälta samman till en kärna måste man kollidera dem med så hög fart att man övervinner de elektriska krafterna som stöter isär dem. För detta behövs höga temperaturer = höga farter på kärnorna och att man har en gas av väte med mycket stor täthet = stor sannolikhet för kollision.

Vissa experiment som gjordes för några år sedan tydde på att man skulle kunna slå samman vätekärnor vid rumstemperatur genom att utnyttja att man kan få mycket höga tätheter av vätgas genom att suga upp gasen i palladium genom elektrolys, se bilden nedan. Senare experiment har inte kunnat bekräfta detta och det anses nu allmänt att det första experimentet inte var riktigt utfört eller möjligen var fusk.

Se vidare Kall_fusion , Cold_fusion och webbsidor under länk 1. Länk 2 är en detaljerad genomgång av skeendet och diskussion om vad som gick fel.



/Lars Gislén/lpe

Nyckelord: kall fusion [8]; pseudovetenskap [11]; vetenskaplig metod [18];

1 http://www.pixe.lth.se/links/search.asp?keyword=cold+fusion
2 http://undsci.berkeley.edu/article/0_0_0/cold_fusion_01

*

  [2856]

Fråga:
Hej! Jag har fått en uppgift av min fysiklärare att förklara varför t.ex. karamellfärg blandar sig med vatten utan att man behöver skak eller röra om. Varför? Den enda ledtråden jag har fått är att jag ska kolla på fysikens grundlagar om jämnvikt. Skulle ni kunna berätta lite om lagarna om jämnvikt? Jag har frågat förr utan svar och hoppas att jag har bättrre tur denna gång! Tack på förhand! /Fredrik
/Fredrik S, Duved, Åre

Svar:
I en vätska rör sig molekylerna hela tiden, de studsar mot varandra. Kunde vi följa en enskild molekyl, skulle den röra sig oregelbundet fram och tillbaka i vätskan. Fenomenet kallas diffusion, och innebär en sorts intern omrörning. Det gör att färgmolekylerna så småningom kommer att fördela sig jämnt i vätskan. 
/KS

*

Energi [3325]

Fråga:
hej, vi har sökt överallt men inte hittat svar på vår fråga. Hur uppnår man den höga temperatur som behövs för att man ska kunna fusionera vätekärnor.
/Mirjam M, Killebäckskolan, S Sandby

Svar:
Det kan ske på flera olika sätt. I en vätebomb är det gamma- och röntgenstrålningen från en atombomb. I en tokamak (som EFDA-JET i England) inducerar man en stark ström i plasmat. Man kan också skjuta in partiklar från en accelerator. Vid JET använder man också radiovågor för upphettningen. Ett annat sätt är att avfyra jättestarka lasrar mot en liten kula som innehåller det tunga vätet. Slutligen finns ett sätt att åstadkomma fusion utan uppvärmning med hjälp av speciella partiklar. Det kallas myonkatalyserad fusion.
/KS/lpe

Se även fråga 1596

Nyckelord: fusion [17];

*

Energi [3695]

Fråga:
Vet man vad ren energi är? Hur gör man när man använder formeln e=mc2? Finns det någon förklaring till varför galaxer börjar snurra?
/Sandra E, Hullbergsskolan, Linköping

Svar:
Om energi, se nedan!

Exempel: Om man i ett fusionskraftverk (sådana finns ännu inte) förbränner 1 kg tungt väte, hur mycket elenergi kan vi då få ut? Ungefär 1% av massan omvandlas till energi. Alltså:

m = 0.01 kg

c = 300 000 000 m/s

E = 0.01 * 300 000 000 * 300 000 000 = 90 000 000 000 000 Joule

Eller 90 000 000 000 000 / 3600 = 25 000 000 000 watttimmar

Eller 25 000 000 000 / 1000 = 25 000 000 kilowattimmar

Eller 25 000 000 / 1000 = 25 000 megawattimmar

Här har vi förutsatt att all energi blir elenergi, vilket säkert inte är fallet.

Om galaxen: galaxen snurrar därför att gasmolnet som bildade galaxen snurrade. "Snurret" bevaras.  
/KS

Se även fråga 3537

*

Energi [4257]

Fråga:
Vad exakt är det som händer när litium bestrålas med neutroner som kommer från en fusionreaktor av tokamak-typ? Det bildas tritium, men hur ser den kemiska formeln ut, varför reagerar litiumen som den gör i detta sammanhang?
/Per R, Buråsskolan, Göteborg

Svar:
Det är fråga om kärnreaktioner, inte kemiska reaktioner. Följande två reaktioner spelar roll:

6Li + n ----> 4He + 3H

7Li + n ----> 4He + 3H + n

I den senare reaktionen slår neutronen sönder Li-kärnan och kan delta i en ny reaktion.
/KS

*

Energi [4981]

Fråga:
Jo, jag gör ett specialarbete om fusion, och jag undrar lite om Stellaratorer. Jag har inte hittat så mycket info om det. Det står mest om tokamakar. Och andra typer av inneslutningar står det också lite om. Vore vänligt om ni kunde ge lite förslag på böcker, artiklar och hemsidor.
/Edwin E, Katedralskolan, Lkpg, Rimforsa

Svar:
Du kan ju börja med länkarna nedan. Det finns en liten artikel i Nationalencyklopedin .
/KS

1 http://www.ipp.mpg.de/E2/hsr/helias.html
2 http://www.ornl.gov/sci/fed/stelnews/

*

Materiens innersta-Atomer-Kärnor [6315]

Fråga:
Jag hade en fråga för några dagar sedan om varför Pons och hans kompanjon borde ha fått dödlig stråldos vid sin kall fusion experiment 1989. Min fråga är egentligen hur mycket (neutron) strålning borde ha utvecklats vid just 4 Watts energiutveckling och varför skulle det vara dödligt. Man måste ju veta hur stor en dödlig stråldos är för att förstå tumregeln.
/Sasan T, Uppsala

Svar:
Det går inte att med enbart uppgiften "4 W" beräkna stråldosen. Man måste veta avstånd, exponeringstid, neutronernas energifördelning, med mera. Neutronstrålning är ett svårt kapitel, också för en garvad strålskyddsfysiker. Vad som är lätt att räkna ut är att 4 W energiproduktion svarar mot 1012 (1000000000000) neutroner per sekund vid deuterium-deuterium fusion.

Närmre detaljer kan man hitta i boken TOO HOT TO HANDLE av Frank Close (1990). Där nämns på sidan 126 att de borde ha fått flera gånger dödlig stråldos. Exakt hur beräkningarna gjorts framgår inte. På sidan 265 finns en grundlig genomgång av de olika reaktionsmekanismerna.

Detaljerad information information finns under länk 1.
/KS

Nyckelord: kall fusion [8];

1 http://en.wikipedia.org/wiki/Cold_fusion

*

Materiens innersta-Atomer-Kärnor [6950]

Fråga:
Vi har en fråga som jag och läraren bråkar om: Vilka partiklar i ett grundämne är de som bestämmer grundämnets egenskaper?
/Lars E, Ekliden, Nacka

Svar:
Ett grundämnes egenskaper bestäms av hur elektronerna är konfigurerade, i synnerhet de yttre elektronerna. Detta beror i sin tur på totala antalet (negativa) elektroner.

Eftersom atomen (normalt) är neutral, måste det finnas lika många (positiva) protoner i kärnan. Man kan säga att det är protonerna som bestämmer, men elektronerna gör jobbet. Så ni har båda rätt !

Ett grundämne är alltså en atom med ett visst antal protoner i kärnan. Kärnfysiker kallar detta Z, medan kemister kallar det atomnummer. Atomnumret bestämmer alltså atomens plats i det periodiska systemet som i sin tur till en stor del bestämmer kemiska egenskaper (se nedanstående bild från Wikimedia Commons). Talet ovanför grundämnesnamnet i varje ruta är atomnumret.

En atomkärna innehåller även ett visst antal neutroner som betecknas N. Totala antalet protoner+neutroner (nukleoner) A=(Z+N) kallas masstal, eftersom kärnans massa uttryckt i massenheter (en massenhet är massan av 1/12 av 12C-atomen) är nära heltalet A.

Ett grundämne i naturen består emellertid ofta av en blandning av atomer med samma Z men olika N, s.k. isotoper (iso=samma, topos=plats, dvs samma plats i det periodiska systemet). Isotoper av ett visst grundämne (givet Z) har alltså samma elektronstruktur och därmed samma kemiska egenskaper. Eftersom isotoper av ett visst ämne har olika antal neutroner, så väger de lite olika. Detta medför att t.ex. lättare isotoper är lite rörligare vid diffusion, vilket används vid anrikning. Man kan med acceleratorer framställa ett stort antal radioaktiva isototoper, som alltså är instabila och sönderfaller till andra mer stabila kärnor.

Ett grundämnes atommassa (även ofta atomvikt) är massan hos en atom av ämnet. För ämnen med flera isotoper avser atommassan ett medelvärde för den naturliga förekomsten. Se List_of_elements_by_atomic_weight för värden och även Formelmassa för hur man räknar på kemiska formler.

Se vidare grundämne , Grundämne och Chemical_element . De individuella grundämnenas egenskaper finns på sajten WebElements .



/KS/lpe

Nyckelord: *kemi [22];

*

Partiklar [7407]

Fråga:
Solen fungerar nästan på samma sätt som en vätebomb, på fusion... Varför exploderar då inte solen som en vätebomb?
/Mattias L, Braås, Växjö

Svar:
Detta är en fundamental fråga. Var de inte rädda för att världshaven skulle explodera när de smällde av den första vätebomben?

Det finns en viktig skillnad. I en vätebomb sker reaktionerna snabbt med starka kärnkrafter. I solen tillkommer en sak, en proton ska omvandlas till en neutron, och det kan inte den starka kärnkraften göra. Det kan bara ske med den svaga växelverkan, och det gör förloppet långsamt. Ytterst handlar det omvandla en u-kvark till en d-kvark. Quark-flavour (som det heter på engelska) kan bara ändras av den svaga växelverkan.
/KS

Se även fråga 4137

Nyckelord: fusion [17];

*

Materiens innersta-Atomer-Kärnor [7988]

Fråga:
Jo jag har en fråga som berör s.k "kall fusion". jag vet att det försök som presenterades av Elektrokemisterna Pons-Fleischmann för ca 10 år sedan tydligen var ogrundat. Pons & Fleischmann menade tydligen att de åstadkomm fusion av typen deuterium-deuterium vid rumstemp, och hävdade att deras reaktion utvecklade en effekt på 4 Watt. Jag har fått höra att en sådan effektutveckling är i sig ett "bevis" för att detta är orimligt då det skulle vara dödande att vistas i närheten av försöksuppställningen vid denna effektutveckling. Vad jag undrar är dels om detta är sant att en effekt om 4 Watt kan vara dödande, och dels unrar jag vad denna effekt har för "natur", är det "strålningseffekt" eller värmeeffekt, eller spelar det ingen roll ? om det nu är strålningen som skulle vara dödande, av vilket slag är den ? Hur ska man tänka då man skall "översätta" utvecklad strålningseffekt till absorberad eller "farlig" effekt ? Kan man se deuterium-deuterium reaktionen som någon slags exoterm reaktion där dessa samtliga 4 Watt utvecklas exotermt. Är det överhuvudtaget vettigt att se denna fusionsreaktion som en exoterm reaktion i någon slags klassisk kemi variant ? /Tack på förhand
/Martin V, Gymn, Uppsala

Svar:
Neutronstrålningen skulle enligt uppgift vara dödlig med hänsyn till effekten, och den tid de vistades i närheten.
/KS

*

Materiens innersta-Atomer-Kärnor [11946]

Fråga:
Jag hade antagit att det var energi som höll samman partiklarna i atomkärnan med varandra. Sedan fick jag reda på att det var någon typ av partikell (minns dock inte namnet.) Jag utgick då ifrån att jag hade haft fel. Nu när vi har läst om kärnfysik på skolan upptäckte jag att jag kanske inte hade så väldigt fel.

När Uran-235 "beskjuts" av en neutron så bildas den ostabila atomen Uran-236. Uran-236 sönderfaller sedan till Krypton-90, Barium-143, tre stycken neutroner OCH energi. Vi läste även att Uran-236 väger mer än Krypton-90, Barium-143 och dess tre neutroner tillsammans. Vikt skillnaden hade alltså blivit energi.

Därav kom jag på att dessa partiklar som håller ihop atomkärnans partiklar är utbytbara mot energi.

Denna teori föll när vi läste vidare om kärnsammanslagning, fusion, då 1/1 H och 7/3 Li två stycken 4/2 He. Litiumet och vätet vägde sammanlagt mer än dom två helium atomerna som bildades.

Om den teori som jag kom fram till hade fungerat så hade litiumet och vätet tillsammans haft en lägre massa än dom två helium atomerna och man skulle alltså inte vinna energi på det. Man skulle förlora energi.

Nu kommer mina frågor:

Stämde det som jag skrev i början om fisson? Hur fungerar det igentligen? Vad har dessa partiklar som håller dom andra partiklarna samman för egenskaper? Är all massa utbytbar mot energi?

Tack för en bra svar på min förra fråga och för det kommande svaret på denna.
/Daniel B, Tråsätra, Åkersberga

Svar:
Ditt resonemang om fission stämmer någorlunda, men behöver en del kommentarer. Uran-236 har en halveringstid på 24000000 år, och är alltså inte särskilt ostabil. Vad som bildas är en exciterad compoundkärna som brukar liknas vid en vätskedroppe som delar sig. Du ger ett exempel på ett fissionsförlopp men det finns många andra. Bland fissionsprodukterna finns kärnor med masstal från 70 till 150.

Det som håller samman en kärna är stark växelverkan. Dess förmedlarpartiklar kallas gluoner.

Energi och massa är ekvivalenta enligt E = mc2. Partikelmassor brukar anges med energienheter. Elektronens massa är 0.511 MeV, eller 0.511 MeV/c2 om man ska vara helt korrekt.
/KS

*

Universum-Solen-Planeterna [14277]

Fråga:
Hej! Jag skulle vilja veta hur det kan komma sig att solens massa kan minska med 4 miljoner ton/sekund? Det är för att det sker fusion i solens inre, men det är en massa/materia som försvinner, men vilken? Jag sa att det var solens massa, men min lärare ville ha ett mer exakt svar! Så jag förstår inte och jag hoppas att ni kan hjälpa mig!
/Maria F, Forssaklack skolan, Borlänge

Svar:
Maria! Uträkningen finns i fråga 13731. 4 miljoner ton per sekund kan tyckas mycket, men kom ihåg att solen totala massa är 2*1030 kg, så det räcker länge!

Det är inte atomer som försvinner utan massa motsvarande den bindningsenergi som frigörs när man gör helium av väte. Detta är bara c:a 0.7% av massan, så vad gäller massan märks det knappast.
/Peter E

Se även fråga 13731

*

Blandat [15996]

Fråga:
Hej! Till vardags säger vi oftast att solskenet har sin grund i att solen förbränner/fusionerar väte till helium. Det är ju också rejält hett i solens inre. Solen lär ju därför inte bestå av neutrala väteatomer. Men har man koll på solens laddningsfördelning? Är den neutral eler är den då och då negativt laddad efter att ha slungat ut partiklar av t ex enbart positiv karaktär? Är det protoner i solens inre eller kan det där finnas fria kvarkar o andra partiklar som inte finns fria i vår kalla värld? Går dessa att studera via ljuset - eller är redan solytan "för kall"?
/Thomas Å, Arlandagymnasiet, Märsta

Svar:
Thomas! Laddningsvariationen måste vara noll eftersom plus och minusladdningar skulle attrahera varandra starkt. Temperaturen i solens inre är c:a 15 miljoner K, och då är alla atomer helt joniserade. Atomkärnor och motsvarande antal elektroner rör sig alltså fritt. Det vi ser från solen är ytan eller fotosfären med en temperatur på 5800 K. Där är atomerna bara delvis joniserade.

Solens nettoladdning är nära noll - både protoner och elektroner slungas ut. Om solen fick för hög laddning skulle den eftersom den elekriska kraften är stark och har oändlig räckvidd dra till sig laddningar så den blev neutral.

Nej, energin i solens inre är inte tillräcklig för att skapa fria kvarkar.
/Peter E

*

Blandat [12105]

Fråga:
Innan jag ställer den egentliga frågan vill jag, helt kort, få veta om jag kan få svar även om man inte är ung och skolelev? Har frågat på liknande sajter förr men aldrig "hört" så mycket som ett knäpp.

Tacksamför svar, Carl
/Carl H, Göteborg

Svar:
Carl!

Resurscentrum för fysik är i första hand till för skolans behov, men vi diskriminerar inte utomstående. Det viktiga är att frågan är "intressant". Bra frågor för Frågelådan har flera av följande egenskaper:

  • Frågan är av rimligt allmänt intresse.
  • Frågan illusterar en fysikalisk princip eller lag.
  • Svaret finns inte tillgängligt genom en enkel sökning i databasen.
  • Frågan behandlar åtminstone ytligt vad man kan kalla fysik, och är inte helt inom en annan specialitet, t.ex. medicin.
  • Stimulerar intresset för fysik hos eleverna.
  • Uppmuntrar eleverna att fundera vidare och med tiden tillägnar sig ett vetenskapligt tänkande (ambitiöst, men vi försöker ).
  • Att frågan har ett vetenskapligt svar som vi kan stå för.

Vi får 20-30 frågor i veckan, och en del frågor tar mycket tid att besvara. Vi måste därför antingen svara mycket kortfattat (t.ex. med en hänvisning till var svaret kan finnas), eller t.o.m. lämna frågor obesvarade. Exempel på frågor som riskerar hamna i denna "strykklass" är:

  • Redan besvarade frågor tillgängliga i databasen.
  • Uppslagsboksfrågor av typen "Vad är jordens diameter?".
  • Räkneuppgifter.
  • Anonyma frågor eller uppenbara "trams-frågor".
  • Vaga eller omfattande frågor, t.ex. "Jag skall skriva ett arbete om energi: berätta allt ni vet om fission och fusion!".

/Peter E

Nyckelord: frågelådan [14];

*

Universum-Solen-Planeterna [12396]

Fråga:
Vad är mörk materia? Kan ni ge något lästips, helst populärvetenskapligt.
/Jonas L, Tegs Centralskola, Umeå

Svar:
Enligt Nationalencyklopedin är mörk materia: materia i universum som inte utsänder mätbara mängder elektromagnetisk strålning och därför inte kan observeras direkt, utan i stället ger sig till känna genom gravitationella effekter på omgivande materia. (mörk-materia )

Vi vet alltså inte exakt vad mörk materia består av men det finns flera kandidater, t.ex.:

  1. Neutriner
  2. Andra svagt växelverkande partiklar (WIMPs)
  3. Materiesamlingar med för liten massa för att fusion skall starta - "misslyckade stjärnor" (MACHOs)
  4. Stora och små svarta hål
Man kan indela mörk materia i baryonisk (3) och icke-baryonisk (1,2) mörk materia. Det finns även förklaringar som innefattar en alternativ gravitationsteori.

WIMP är en term inom astro- och partikelfysik. Det är en akronym för weakly interacting massive particles, eller ”svagt växelverkande massiva partiklar”. Dessa partiklar föreslås vara en spöklik form av materia, som vi inte direkt kan se eller enkelt mäta, och som bara ger sig till känna genom sin gravitation och svaga växelverkan.

En alternativ, om än idag något mindre plausibel, teori för den mörka materien är MACHO:s. MACHO är en akronym av massive astrophysical compact halo object, som är mörka eller mycket ljussvaga objekt som svarta hål, neutronstjärnor och bruna dvärgar etcetera som antas förekomma i galaxers halon och som utgör en form av mörk materia.

Börja med att läsa artikeln i NE. Sök sedan med en sökmotor, t.ex. länk 1, det finns många bra dokument även på svenska, t.ex. Mörk materia .

Wikipedia-artiklarna Dark_matter och Mörk_materia är också aktuella och bra.
/Peter E

Nyckelord: mörk materia [17]; WIMPs [3];

1 http://www.google.com/search?&q=m%C3%B6rk+materia

*

Universum-Solen-Planeterna [12547]

Fråga:
Hur kan solen brinna utan att det finns syre i rymden?
/Tuva W, Årstaskolan, Stockholm

Svar:
Därför att solen "brinner" inte. Om all energi som strålar ut från solen skulle komma från vanlig förbränning (oxidation, dvs kemiska reaktioner med syre) så skulle solen inte kunna skina mer än några tusen år. Vi vet att den haft i stort sett samma ljusstyrka i 5 miljarder år. Innan början på 1900-talet var detta ett stort vetenskapligt problem.

Nu vet vi att solens energi kommer från sammanslagning av väte till helium. Fyra vätekärnor (protoner) ger en heliumatom, några partiklar till plus en väldig massa energi. Solen kan fortsätta så i 5 miljarder år till.

Proton-protonkedjan är en av flera fusionsreaktioner genom vilka stjärnor omvandlar väte till helium, det främsta alternativet är CNO-cykeln. Proton-protonkedjan dominerar i stjärnor av solens storlek eller mindre. Se vidare Proton–proton_chain_reaction och nedanstående bild.



/Peter E

Se även fråga 10658 och fråga 3930

Nyckelord: solens energiproduktion [9];

*

Energi [12726]

Fråga:
det finns ju ett samband mellan energi och materia. altså är materia i lekmansterm sammanslagen energi. kan man utvinna energin, och omvända, materian till energi? och i så fall, hade det kunnat finnas 100% energi och 0% materia i hela universum?

kanske dåligt formulerat, men du förstår nog.
/fredrik l, naturhumanistiska, helsingborg

Svar:
Fredrik! Alldeles i början på vårt universums existens - precis efter Big Bang - tror vi att det bara fanns energi, ingen materia! Allt eftersom universum expanderade och "svalnade" omvandlades energin till den materie världen består av nu i flera steg.

Kan vi nu gå åt andra hållet, dvs omvandla massa till energi? Jodå, det går alldeles utmärkt nere på kärnnivå - genom att antingen slå ihop lätta atomkärnor till tyngre (fusion) eller klyva tyngre atomkärnor (fission) kan vi omvandla en del av kärnmassan till energi enligt Einsteins formel (E=mc2).

Hur det går till kan vi se på bilden nedan (från Nationalencyklopedin ). Kurvan visar hur atomkärnans potentiella energi ändrar sig när atomkärnans massa ökar. Vi ser att lätta kärnor som deuterium (D, "tungt väte") har hög potentiell energi, så när man slår ihop dem till helium (4He) kan man utvinna energiskillnaden (den skära stapeln). Om vi tvärtom klyver mycket tunga kärnor som uran (U) så att vi bildar två lättare kärnor (med massa i närheten av kurvans minimum), frigörs också energi (den gröna stapeln).

Den potentiella energin i bilden motsvaras av kärnornas inre bindningsenergi - ju högre den potentiella energin är, desto lägre är bindningsenergin per massenhet.

Läs mer: Slå upp "universum", "fusion", "fission" och "bindningsenergi" i t.ex. Nationalencyklopedin ....



/Margareta H

Nyckelord: massa-energi [1]; fusion [17]; fission [15]; bindningsenergi [23];

*

Energi [12940]

Fråga:
1 Hur långt har man kommit i forskningen om miljövänliga bilar?

2 Vilka miljövänliga drivmedel finns förutom etanol och biogas?

3 Vad är det som gör att man inte kommit längre i användningen av miljövänliga bilar?

4 Jag har hört ett rykte om att man skulle kunna driva bilar på vatten, är det sant?

5 Om elmotorer ska utvecklas, var ska vi då få elen ifrån?
/Hanna H, Katrinelund skola, Sundsvall

Svar:
1 Det stora problemet är inte att uppfinna nya bilar utan att bilidustrin skall vilja tillverka dem. Bilindustrin drivs av ekonomi, och så länge dom gör goda förtjänster på traditionella bilar, så fortsätter dom med det. Det finns redan flera konstruktioner som är betydligt bättre.

2 Vad man kan förbättra är dels bränslet, och där är etanol och biogas två exempel. Väte är också en möjlighet, men det kräver en helt annan motor. Sedan har man provat s.k. hybridbilar som har en vanlig motor för långa sträckor. Motorn laddar även ett batteri, som med hjälp av en elmotor kan driva bilen inne i städer. Vanliga elbilar finns också, men de har mycket kort räckvidd eftersom batterier blir stora och tunga.

3 Se ovan.

4 Nej, vatten innehåller inte energi man kan utnyttja för en motor. Möjligen kommer detta av att vatten innehåller deuterium, som kan användas för fusion. Om man överhuvudtaget lyckas bygga en fusionsreaktor inom 100 år så kommer den definitivt inte gå in i en bil!

5 Solenergi, vattenkraft, vindkraft eller kärnenergi. En intressant möjlighet är att med elektricitet från någon av dessa källorna producera vätgas och syrgas genom elektrolys av vatten. Dessa gaser kan sedan i med hjälp av en s.k. bränslecell producera elektricitet som kan driva bilen, se länk 1. Mycket detaljerad information och länkar finns på Fuel_cell . Observera emellertid att vätgasen inte är en energikälla utan en energibärare eftersom vätgasen måste framställas med elektrolys, vilket kostar mycket energi.
/Peter E

Nyckelord: *miljöpåverkan [14]; bränslecell [3];

1 http://fy.chalmers.se/ef/Brcell.htm
2 http://auto.howstuffworks.com/hybrid-car.htm

*

Materiens innersta-Atomer-Kärnor [12960]

Fråga:
Hej, jag håller på med mitt projektarbete som handlar om kärnfysik och energiutvinning. Jag har några frågor. Idag har vi ju ännu inte något elproducerande fusionskraftverk och jag undrar hur ett sådant skulle kunna se ut i framtiden?. I ett kärnkraftsverk så värmer man ju upp vatten genom klyvning som senare går genom en turbin och till en generator och då har man ström. Vattnet fungerar ju också som moderator och dämpar strålningen i ett kärnkraftsverk. Men i ett framtida fusionskraftverk skulle väl inte vatten vara av samma betydelse?

Borde man inte i ett framtida fusionskraftverk kunna omvandla energin från fusionen direkt till elektrisk energi via generator istället för att gå omvägen via vatten och en turbin. Eller måste man värma upp vatten och gå den vägen för att kunna tillgodogöra sig fusionsenergin?

Jag har en annan fråga också och undrar hur spontan fission i naturen fungerar. Skulle det inte kunna uppstå en kedjereaktion typ som i en atombomb?
/Adam Ö, Falu frigymnasium, Falun

Svar:
Standardkonstruktionen på en fusionsreaktor är en torusformad (bilringsformad) inneslutning med ett magnetfält. De laddade partiklarna i den heta plasman kommer alltså inte åt röret som bildar torusen. Neutronerna kan däremot ta sig ut, och det är dessa som kan transportera energin ut från reaktorn.

Man har lyckats med vad som kallas "theoretical break even" i JET, EFDA-JET . Detta innebär att man fått ut lika mycket energi som man satt in (lika mycket energi kommer ut från fusionen som man sätter in för att värma plasman). Problemet är att energin finns i form av neutroner med hög energi, och problemet man skall lösa med nästa generation fusionsreaktor (ITER, EFDA , se nedanstående bild) är att ta vara på den energin. Hur detta skall ske och vilka material man skall använda vet man inte ännu. Se ovanstående länkar, Fusion_power och nedanstående sökning för mer information om fusion.

Det fordras mycket speciella förhållanden för att en kedjereaktion skall vara möjlig. Man har hittat plutonium i en uran-gruva i Afrika (Oklo, Gabon), och eftersom plutonium är ganska kortlivat har plutoniet antagligen bildats i en naturlig reaktor, se En naturlig kärnreaktor: Oklo och Natural_nuclear_fission_reactor . Naturligt uran innehåller bara 0.7% av den klyvbara U-235, så naturliga kedjereaktioner är mycket ovanliga.

I en vanlig lättvattenreaktor har man 3-5% U-235. Det är då precis möjligt att vidmakthålla en kedjereaktion. För ett kärnvapen behöver man nära 100% U-235, så en atombomb kan inte uppstå naturligt.



/Peter E

Nyckelord: fusion [17]; fission [15];

Avancerad sökning på 'fusion' i denna databas

*

Universum-Solen-Planeterna [12974]

Fråga:
En stjärna med stor massa har ju kort livstid på grund av att fusionen har ett snabbare förlopp än i en liten. Innebär inte detta att temperaturen i stjärnans centrum också är högre än i en liten stjärna?
/Ingrid T, Göteborg

Svar:
Ingrid! Det är korrekt att en tyngre stjärna har högre temperatur i en större region i centrum. Detta gör att väteförbränningen blir effektivare, och sjärnan får kortare livslängd än en lättare stjärna. Detta reflekteras ju även i yttemperaturen - massiva stjärnor är ju inte bara ljusare, deras yttemperatur är också högre (se fråga 12771).
/Peter E

Se även fråga 12771

1 http://fy.chalmers.se/~numa/VL/stellarmasses.pdf

*

Energi, Materiens innersta-Atomer-Kärnor [12978]

Fråga:
Jag undrar hur fusion kan vara en outtömlig energikälla. Vad jag har förstått ska man kunna utvinna deuterium av vanligt vatten. Men borde inte vattnets kretslopp kunna rubbas om man omvandlar vätet i vatten till helium. Vatten är ju någonting vi inte kan leva utan och borde man inte vara rädd om det.

Undrar en annan sak också. Deuterium-tritium ska ju vara den reaktion som ger mest energi. Men skulle man inte kunna slå ihop tritium-tritium vid fusion oc vad får man isåfall då?

Jag läste någonstans att man skulle kunna använda sig av någon form av Helium-3 istället för tritium som bränsle och på så sätt slipper att neutroner åker iväg och fastnar i reaktorns väggar. Hur skulle detta fungera och hur skulle reaktionsformeln för detta se ut?
/Adam Ö, falu fri, Falun

Svar:
Vatten innehåller väldigt lite deuterium, så även om man skulle kunna ta bort allt deuterium så skulle det inte märkas.

Det är rätt att deuterium+tritium är den mest lovande reaktionen - hög sannolikhet och mycket utvecklad energi, se nedanstående animering. Man vinner egentligen inget med tritium+tritium. Eftersom man måste framställa tritium ur litium-6:

6Li + n --> 3H + 4He

är det förekomsten av litium som är begränsningen, se länk 1. Länk 2 behandlar en alternativ metod: produktion av tritium från He-3.

I princip skulle reaktionen

3He + d --> 4He + p

fungera lika bra som tritiumreaktionen

3H + d --> 4He + n

Med fusion i en magnetisk inneslutning är det emellertid en fördel att den lätta reaktionsprodukten är oladdad. Den kan då ta sig ut ur den magnetiska inneslutningen och på så sätt transportera ut energi så den kan tas tillvara. Eventuellt kan neutronen även användas för att producera tritiumet. Se vidare nedanstående frågor.

Låt oss avslutningsvis titta på vilka metoder man kan använda för att åstadkomma fusion. Man behöver

  1. Atomer som kan genomgå en lämplig fusionsreaktion (hög reaktionssannolikhet och stor energiutveckling)
  2. Tillräckligt hög densitet
  3. Tillräckligt hög temperatur (rörelseenergi som kan övervinna coulombrepulsionen)
Inneslutningen kan ske på följande sätt
  1. Gravitation (det krävs en massa av storleksorningen solens, så detta är bara aktuellt för stjärnor)
  2. Tröghet (höja temperaturen så snabbt med t.ex. laser eller accelerator så att atomerna inte hinner flytta på sig)
  3. Magnetisk inneslutning (Tokamak)


/Peter E

Nyckelord: fusion [17];

1 http://www-fusion-magnetique.cea.fr/gb/en_savoir_plus/lithium/lithium.htm
2 http://apt.lanl.gov/system.html

*

Energi, Materiens innersta-Atomer-Kärnor [13191]

Fråga:
Varför slår man inte ihop två deuterium till helium?
/Veckans fråga

Ursprunglig fråga:
Jag håller på med ett arbete om fusion och har ett par frågor. Varför inte använda flera deuterium och slå ihop? På så sätt får man ingen "överbliven" neutron som man får vi "vanlig" D-T fusion. Sen så undrar jag också hur det kommer sig att en neutron kan komma ut ur det starka magnetfältet som håller plasman och träffa innerväggarna? Tack på förhand.
/Viktor W, Johanskyttes skola, Älvsjö

Svar:
Viktor! Du har rätt i att d+d kan ge 4He, och det är denna man helst vill ha eftersom den är mest bunden (och ger därmed mest energi). Det finns emellertid två problem.

Det ena är att man får nästa inga 4He i d+d reaktionen, man får nästan bara 3He+n och 3H+p. Detta beror på att om man slår ihop två deuteroner så bildas 4He i ett högt exciterat tillstånd (över 20 MeV). Detta tillstånd är obundet och sönderfaller med utsändande av en proton eller en neutron. Det finns en mycket lite chans att det sönderfaller genom att skicka ut ett gamma, men eftersom den elektromagnetiska växelverkan är så svag är detta en nästan försumbar gren.

Det andra problemet är att man vill ha en neutron ut. Neutronen är oladdad, så den kan ta sig ut genom magnetfältet. Laddade partiklar hålls innestängda av magnetfältet. Neutronerna är alltså bärarna av energin man vill ha ut ur fusionsreaktorn. Hur man sedan skall ta vara på denna energi är ännu inte klart. Det är detta man vill finna ut med ITER (International Thermonuclear Experimental Reactor), se EFDA och International Thermonuclear Experimental Reactor (ITER) .

Se även kall fusion .
/Peter E

Nyckelord: fusion [17];

*

Universum-Solen-Planeterna [13313]

Fråga:
Varför expanderar en stjärna när bränslet i centrum tar slut?
/Veckans fråga

Ursprunglig fråga:
När vätet i centrum av en stjärna av solens storlek är slut, och helium bildats, expanderar solen till en röd jätte. Stålningsenergin avtar, den utåtriktade kraften minskar således, medan gravitationen kvarstår. Men varför expanderar då stjärnan?
/Christina E, Christian 4:s Gymnasium, Kristianstad

Svar:
Energiproduktionen fortsätter i ett skal kring den kontraherade He-kärnan. Om stjärnan har tillräckligt stor massa och når en temperatur på c:a 100 miljoner grader kommer fusion av He till C att komma igång i centrum. Det är alltså inte riktigt att effekten (energiproduktionen per tidsenhet) minskar - den ökar i själva verket. I HR-diagrammet nedan kan man se att röda jättestjärnor har HÖGRE ljusstyrka än huvudseriestjärnor av solens storlek. Omfördelningen av massa och energiproduktion tvingar stjärnan att expandera för att den skall bevara den hydrostatiska jämvikten. Expansionen av de yttre delarna orsakar en temperatursänkning, varför stjärnan blir rödare. För detaljer se What will happen to the Solar System in the future och nedanstående länk.



/Peter E

Nyckelord: stjärnors utveckling [15]; HR-diagram [3];

1 http://media.pearsoncmg.com/bc/bc_bennett_essential_2/etext/bodymatter/03/5801.html

*

Materiens innersta-Atomer-Kärnor [13570]

Fråga:
Vi ska ha en debatt i skolan och jag skulle behöva få reda på vilka positiva (ren fysik inte etik och moral) saker det finns med att tillverka/användning av kärnvapen.
/Edina k, trollboda skolan, hässelby

Svar:
Edina! När det gäller kärnvapen är det svårt att bortse från etik och moral. Det enda positiva är att kärnvapen är så förfärliga att de är omöjliga att använda i krig. De har visserligen använts en gång (Japan 1945), men de flesta anser i dag att detta inte var nödvändigt. Tack vare terrorbalansen (MAD = Mutual Assured Destruction, ömsesidig säker förstörelse) undveks en väpnad konfrontation mella NATO och Sovjetunionen under det s.k. kalla kriget.

Kän man använda kärnvapen till något bra? Mja, man har tänkt sig små kärnexplosioner för att driva fram en raket ute i rymden. Projektet las emellertid ner. Man har använt kärnvapen för studier av atomkärnor, men det är dyrt och krångligt, speciellt som detektorerna förstördes på mindre än en sekund!

Det har kommit upp ett par filmer om hur man räddar jorden från en kollision med småplaneter/kometer (t.ex. Deep Impact) med hjälp av kärnladdningar. Realismen i detta betvivlar jag dock, framför allt för att tiden från upptäckt till kollision är för kort.

Viktigast är säkert att genom utvecklingen av kärnvapen så lärde vi oss mycket om atomkärnan och den enorma energi som finns tillgänglig i tunga (genom fission) och lätta (genom fusion) kärnor. Kärnkraftverk utvecklades och ger fortfarande ett stort bidrag till vår energiförsörjning utan produktion av CO2.

Slå upp de kursiva orden i Nationalencyklopedin . Se även nedanstående länk.
/Peter E

Nyckelord: kärnvapen [16];

Avancerad sökning på 'kärnvapen' i denna databas

*

Materiens innersta-Atomer-Kärnor [13679]

Fråga:
Verkan av kärnvapen
/Veckans fråga

Ursprunglig fråga:
Hur länge var marken radioaktiv i Hiroshima och Nagasaki efter atombombningarna? Hur går man till väga vid en strålningskatastrof, finns det något sätt att få bort strålningen ur marken?
/Jesper E, Karro, Örebro

Svar:
Kärnvapen är en benämning för olika typer av vapen vars sprängladdning får sin energi från fission eller från olika kombinationer av fission och fusion, till skillnad mot konventionella vapen vars sprängladdningar får sin energi från kemiska processer, se Kärnvapen .

Man kan inte svara exakt på den första frågan: det bildas nuklider med både kort och mycket lång halveringstid. Efter något år är nvåerna nere till i storleksordningen den naturliga bakgrundsstrålningen, men med känsliga detektorer kan man detektera strålning som härrör från ett kärnvapen under mycket lång tid. Efter 6 dagar uppskattar man att endast 10% av radioaktiviteten fanns kvar.

Det är svårt att sanera ett område som blivit kontaminerat av radioaktivt material. Radioaktivt nedfall kan man i princip få bort genom tvättning och bortforslande av det översta jordlagret. Inducerad radioaktivitet (se nedan) är nästan omöjligt att göra något åt såvida man inte forslar bort precis allt.

Låt oss passa på och gå igenom de verkningar man får av ett kärnvapen:

Tryckvåg

Den kraftiga explosionen skaper en tryckvåg som gör mycket stor skada. Än värre blir skadan eftersom den första tryckvågen utåt följs av en nästan lika kraftig inåt. Den senare skapas av det undertryck som uppstår när luften värms upp och därmed stiger.

Värmestrålning

Värmestrålning från det upphettade plasmat som bildas av explosionen. Värmestrålningen sätter eld på allt brännbart nära explosionsplatsen. För mycket stora kärnvapen (vätebomber) är värmestrålning den dominerande skadeverkan.

Omedelbar joniserande strålning

I fissionsprocessen (klyvning av tunga atomkärnor) bildas neutroner och gamma/röntgenstrålning. En del av neutronerna (det bildas 2-3 per kärnklyvning) går åt till att hålla kärnklyvningen vid liv (drygt 1 neutron), medan resten flyger ut med hög hastighet till dom träffar något, se inducerad radioaktivitet.

Denna omedelbara strålning ger en skadlig stråldos dos till den som befinner sig nära. För små kärnvapen är detta den dominerande skadeverkan (neutronbomber).

Elektromagnetisk puls

Elektromagnetisk strålning som inducerar stömmar i ledningar och förstör elektronik. För detaljer se fråga 13095 nedan.

Radioaktivt nerfall

De som ger energin till explosionen är ett snabbs förlopp (kedjereaktion) av klyvning av 235U eller 239Pu. Klyvningen induceras av neutroner som bildats i en tidigare kärnklyvning. Resultatet av kärnklyvningen (fission) är två medeltunga kärnor (kallade klyvningsprodukter), 2-3 neutroner och mycket energi. Klyvningsprodukterna är oftast radioaktiva, och de faller ner efter ett tag och utgör en fara för omgivningen - framför allt i vindriktningen.

Inducerad radioaktivitet

De neutroner som kommer ut vid explosionen träffar meterial på marken och kan förorsaka kärnreaktioner i detta. Material i omgivningen kan då bli radioaktivt. Denna aktivering av omgivningen har i Hiroshima och Nagasaki använts för att bestämma hur hög stråldos personer på olika platser utsatts för. Denna kunskap, tillsammans med statistik på sena skador (mest cancer) har givit oss goda kunskaper om skadeverningarna av joniserande strålning, åtminstone var gäller relativt höga doser.

Hur länge finns risker?

Varför kunde man då mycket kort efter bomberna i Hirishima och Nagasaki flytta tillbaka, medan Tjernobyl fortfarande är alldeles för kontaminerat för att man skall kunna vistas där? Skillnaden är dels att det kom ut mycket större mängd (i ett kärnvapen finns några tiotals kg klyvbart material, i ett kärnkraftverk hundratals ton) klyvningsprodukter i Tjernobyl och de var i medeltal mer långlivade. I ett kärnvapen produceras klyvningsprodukter under en mycket kort tid, och ganska få av dessa är långlivade. I ett kärnkraftverk pågår kärnklyvningen under lång tid, varvid de långlivade nukliderna finns kvar medan de kortlivade sönderfaller.

Se vidare länk 1 och länk 2 under 'Hiroshima and Nagasaki Health Effects'. Nuclearfiles.org innehåller mycket information om kärnvapen. Bilden nedan kommer från denna sajt.



/Peter E

Se även fråga 13095

Nyckelord: kärnvapen [16]; Tjernobyl [12]; Hiroshima/Nagasaki [4];

1 http://www.hindu.com/thehindu/2001/09/06/stories/08060003.htm
2 http://hps.org/publicinformation/ate/cat25.html#141

*

Materiens innersta-Atomer-Kärnor, Universum-Solen-Planeterna [13731]

Fråga:
Solens massa minskar med 4 miljoner ton varje sekund. Kan du förklara det?
/Veckans fråga

Ursprunglig fråga:
hejsan . Jag är lite av en nybörjare inom fysiken men jag har på min fritid fifflat på en liten uträkning på hur mycket solen kommer generera i energi tills alla väteatomer är omvandlade. Nu satt jag och läste på er sida och såg en uträkning på hur mycket en atom genererar genom fusion. detta räknades i MeV? Kan jag få en förklaring till detta det hade gjort min uträkning lättare..
/Kenny j, Galären, Karlskrona

Svar:
Fyra väteatomer bildar en heliumatom under frigörande av 26.7 MeV, se fråga 10658 .

Vad behöver vi mer för att räkna ut svaret på din fråga? Vi behöver solens massa och andelen väte (låt oss säga när solen skapades). Solens massa är 1989100*1024 kg (Planetary Fact Sheets ). Andelen väte i den unga solen är lite mer osäkert - det beror på om gasen som bildade solen kom direkt från big bang eller om den redan varit i en heliumproducerande stjärna. Låt oss anta 75% väte som en rimlig gissning. Vi har alltså 0.75*1989100*1024=1.5*1030 kg väte. Molvikten är 1 g, så vi har 1.5*1030/0.001=1.5*1033 mol väte.

Avogadros konstant (antalet atomer per mol av ett ämne) är 6.022*1023 mol-1 (Physical Reference Data ). Vi kan räkna ut antalet väteatomer som

1.5*1033*6.022*1023=9.0*1056

Fyra väteatomer gav 26.7 MeV så totala energiutvecklingen om vi transformerar allt väte till helium blir

26.7*9.0*1056/4 = 60*1056 MeV

Om vi förvandlar till J får vi energin

60*1056*106*1.602*10-19 = 97*1043 J

Detta är alltså solens energipotential om allt väte kan förvandlas till helium. Som sådant är detta värde kanske inte så intressant, men låt oss räkna ut hur länge solen skulle kunna lysa med sin nuvarande styrka. Solens effekt (luninositet) är enligt Planetary Fact Sheets 384.6*1024 J/s (W). Maximala åldern blir då

97*1043/(384.6*1024) = 0.25*1017 sekunder =
= 0.25*1017/(60*60*24*365.24) = 80*109 år = 80 Ga = 80 miljarder år

Nu är det emellertid en våldsam överskattning att allt befintligt väte förvandlas till helium. En rimligare uppskattning är 10%, och då skulle solens maximala livslängd vara c:a 8 miljarder år, vilket är nära de 10 miljarder år man får fram med mer detaljerade beräkningar.

Solens massförlust

Om den producerade energin per sekund är 384.6*1024 J (se ovan) hur mycket lättare blir solen då varje sekund. Vi använder E = mc2 och får massförlusten per sekund m till

m = E/c2 = 384.6*1024/(3*108)2 = 4270 000 000 kg

vilket är ungefär 4 miljoner ton! Denna massa försvinner naturligtvis inte - totala massan/energin måste bevaras - utan den sprids ut i universum av solstrålningen. Om strålningen träffar något - t.ex. en planet - kommer massan hos planeten att öka lite, lite grann.
/Peter E

Nyckelord: solens energiproduktion [9];

1 http://fragelada.fysik.org/index.asp?keyword=stj%E4rnors+utveckling
2 http://www.linnaeus.uu.se/online/fysik/makrokosmos/solcykeln.html

*

Universum-Solen-Planeterna [13738]

Fråga:
min fråga gäller stjärnorna, jag undrar om ni vet hur man beräknar mini-massan i stjärnor innan fusionen sätter igång, i tex en formel eller nåt, en formel skulle uppskattas. sen har jag en annan, det gäller en stjärnas livstid, hur man räknar ut den med hjälp. helst skulle jag villa kunna räkna in dom olika kraft konstanterna i dessa beräkningar men går det inte gör det inget
/timmy j, rudbeck, örebro

Svar:
använder man inte stora bokstäver i början av meningar längre?

Det finns inte en formel som säger vad den minsta massan är för att fusion skall starta. Värdet är c:a 1/10 solmassa, men man måste använda ganska komplicerade simuleringsprogram för att beräkna värdet. I de allra lättaste stjärnorna kommer fusion av deuterium (tungt väte) igång, men inte av vanligt väte. Gränsen mellan stjärnor och planeter (bruna dvärgar) är ganska flytande och kontroversiell (se länk 1).

Vad gäller en grov uppskattning av en stjärnas livstid se 13731 nedan. För mer om stjärnors utveckling se Stellar_evolution .
/Peter E

Se även fråga 13731

Nyckelord: stjärnors utveckling [15];

*

Materiens innersta-Atomer-Kärnor [13744]

Fråga:
Radon och naturlig radioaktivitet
/Veckans fråga

Ursprunglig fråga:
Jag är Högskolestudent inom samhällsvetenskap och har en fråga angående radon.

Jag undrar hur den fysiska / kemiska processen ser ur när radondöttrar bildas. Bildas de av sig självt eller genom fotolys? Vad har radonet samt "dess döttrar" för fysiska beteckningar? Faller dessa sönder i yttligare beståndsdelar och i så fall hur osv.
/Robin S, Göteborg

Svar:
Naturlig radioaktivitet

Alla grundämnen tyngre än helium har bildats i stjärnor genom fusion och vid slutet av stjärnans liv kastats ut i rymden. Dessa grundämnen kommer sedan ingå i materialet som bildar nästa generation stjärnor och planetsystem.

Många av de nuklider (en nuklid är en kärna med ett visst antal neutroner och protoner) som bildas på detta sätt är radioaktiva, och sönderfaller med en karakteristisk halveringstid. Solsystemet är c:a 4.5 miljarder år gammalt, så endast nuklider med halveringstider av storleksordningen miljarder år kan finnas kvar i dag.

De viktigaste av dessa ursprungliga radioaktiva ämnena är 40K (halveringstid 1.3 miljarder år), 235U (0.71 miljarder år), 238U (4.5 miljarder år) och 232Th (14 miljarder år),

De tre tunga ämnena ovan är urspunget till tre naturligt förekommande radioaktiva kedjor med en serie a- och b--sönerfall, se nedan.


U-235 serien


U-238 serien


Th-232 serien

Sedan finns även ett antal radioaktiva ämnen som ständigt bildas av den kosmiska strålningen. De viktigaste är 14C (5730 år), tritium (3H, 12.3 år) och 7Be (53 dagar).

Radon

I alla tre serierna ovan finns ädelgasen radon (Rn) i sönderfallskedjan. Den med längst halveringstid är 222Rn (3.8 dagar) i U-238 kedjan. Låt oss begränsa diskussionen till denna. När serien av sönderfall kommit till 222Rn kan, eftersom Rn är en gas, den radioaktiva nukliden ge sig iväg från uran-malmen och eventuellt upp till jordytan. Om radonet kommer in i ett hus och ansamlas där kan det vara mycket farligt.

En a-strålande källa är normalt inte särskilt skadlig, eftersom a-partiklarna stoppas av det yttersta hudlagret. Radon är emellertid en gas och kan komma in i lungorna. Om radonet sönderfaller där, kommer dotterkärnan och hela serien av döttrar att stanna i lungorna och ge upphov till mycket koncentrerade och skadliga strålskador. Radon-döttrarna är alltså alla nuklider som ligger efter radon i serien.

Den viktigaste effekten äv denna bestrålning är cancer. Mängden radon i hus varierar mycket över Sverige. Förekomsten är i första hand beroende av husets konstruktion och förekomsten av uran i berggrunden. Gränsvärdet för bostäder i Sverige är 200 Bg/m3, se Rikt- och gränsvärden för radon .

Det är svårt att uppskatta antalet cancerfall som orsakas av radonbestrålning (rökning är en mycket större orsak), men den senaste siffran jag sett är 800 personer per år i Sverige. Men, som sagt, denna siffra är mycket osäker.
/Peter E

Nyckelord: radon [4]; naturlig radioaktivitet [3]; strålning, faror med [26]; radioaktivitet, sönderfallskedja [7];

1 http://www.physics.isu.edu/radinf/natural.htm
2 http://hyperphysics.phy-astr.gsu.edu/hbase/nuclear/radser.html

*

Blandat [13752]

Fråga:
Vad är luktens hastighet?
/Veckans fråga

Ursprunglig fråga:
Vad är den fria medelväglängden för en luftmolekyl? Är det rimligt att en luftmolekyls fart är ca 300m/s? Frågan kommer sig av att vi i klassen diskuterar hur lukten från t ex en parfymflaska sprids i ett rum.
/Therese N, Högskolan i Gävle, Gävle

Svar:
Therese! Ja, vad är luktens hastighet? Ljudets hastighet är den hastighet med vilken tryckförändringar fortplantar sig i luft, se fråga 12639. Låt oss se vad resultatet blir om vi förklarar luktspridning med diffusion. Enligt Nationalencyklopedin är

diffusion: spontan materialtransport eller utbredning av ett ämne, orsakad av slumpvisa förändringar i egenrörelserna hos ämnets atomer eller molekyler (diffusion ).

Ni har helt rätt i att diffusion beror av den fria medelväglängden och molekylernas medelhastighet.

Fria medelväglängden är bra utredd under länk 1 nedan. Där finns även en kalkylator. Värdet för normaltrycket 760 mmHg och 300 K är 0.92 mikrometer, dvs c:a 10000 gånger molekylens diameter (vi har approximerat molekyldiametern till 0.1 nm).

Molekylernas medelhastighet v (rms) kan fås ur

m*v2/2 = 3*k*T/2 dvs

v = (3*k*T/m)1/2

m är molekylens massa, T är absoluta temperaturen och k=1.3806505*10-23 J K-1 är Boltzmanns konstant, se Physical Reference Data . Vi behöver även en atomär massenhet u = 1.66053886*10-27 kg från samma källa.

För luft (molmassa 28.8) och temperatur 300 K blir medelhastigheten (rms, dvs medelvärdet av v2) ungefär 510 m/s med ovanstående formel. Kalkylatorn under länk 1 ger naturligtvis samma värde för v(rms), men det vi egentligen behöver är medelvärdet av v som blir 470 m/s.

Molekylerna transporteras med vad som brukar kallas "random walk". För denna ges den tillryggalagda sträckan d av

d = l*N1/2

där l är fria medelväglängden och N antal kollisioner, se länk 2. Den totala vägen molekylen färdas är

N*l = v*t

där v är medelhastigheten och t tiden. Insättning i formeln för d ger då

d = l*(v*t/l)1/2 = (v*t*l)1/2

Vi ser alltså att den tillryggalagda sträckan inte som brukligt är proportionell mot tiden utan mot roten ur tiden. Sätter vi in ovanstående värden får vi

d = (470*t*0.92*10-6)1/2

Ovanstående uttryck har plottats med MatLab, se nedan. Man behöver alltså vänta i mer än en halv timme för att den typiska molekylen skall ha kommit en meter bort! Medelhastigheten (högra panelen) är alltså nere i 0.2 mm/timme efter knappt en timme! Observera att diagrammen beskriver transporten av en medelmolekyl. De molekyler som har "tur" kommer fram betydligt snabbare. För en parfymmolekyl är säkert molmassan och diametern mycket större, vilket medför en ännu långsammare doftspridning!

Man kan då fråga sig varför kvinnor använder parfym när mannen endast kan attraheras när han i princip redan är infångad. Antingen kan kvinnor inte fysik - vilket inte är sant - eller så finns det en annan effekt än diffusion som sprider doften. Små vindpustar som orsakas av att något rör sig är mycket mer effektiva för att transportera dofter!



/Peter E

Se även fråga 12639

Nyckelord: diffusion [1];

1 http://hyperphysics.phy-astr.gsu.edu/hbase/kinetic/menfre.html
2 http://spiff.rit.edu/classes/phys440/lectures/walk/walk.html

*

Elektricitet-Magnetism [13770]

Fråga:
Hej, om man har en kopparledning och skickar på lite el, hur kommer då elektronerna förbi kopparatomerna,om de hoppar mellan atomerna som jag har hört? Då undrar jag om det är samma elektroner eller om de byts ut vid varje atom?
/Pär T, komvux, Karlstad

Svar:
Pär! Problemet är ganska likt det i svaret om diffusion, se diffusion , men elektronerna hindras inte bara av varandra utan även av atomerna. Man får skilja på elektronernas momentana hastighet, deras drifthastighet och strömmens hastighet. Vad gäller de första två se fråga 9549 . Drifthastigheten (diffusion) är alltså mycket låg, den momentana hastigheten ganska hög och strömmens hastighet i princip ljushastigheten. Det kan tyckas konstigt att strömmens hastighet är så mycket högre än drifthastigheten, men du skall föreställa dig det så här:

Spänningen (det elektriska fältet) över ledaren utbreder sig med ljushastigheten. När elektronerna känner det elekriska fältet tvingas deras medeldrifthastighet att bli i fältets riktning - utan fält är drifthastigheten helt slumpmässig. Eftersom alla elektroner påverkas av fältet nästan samtidigt, kommer stömmen igång mycket snabbt i hela ledarens längd.
/Peter E

Nyckelord: elektrisk ström, hastighet [4];

*

Universum-Solen-Planeterna [13916]

Fråga:
Vad är en kvasar?
/Veckans fråga

Ursprunglig fråga:
Hej! Vi har sett en film där dom påstår att massiva svarta hål matar kvasarer. Och i en bok har vi läst att en kvasar är ett komprimerat galaxcentrum med massor av stjärnor. Så nu undrar vi om du kan förklara så enkelt som möjligt hur allt detta hänger ihop. För hur ska svarta hålen kunna mata den komprimerade galaxkärnan? Svarta hål ska ju snarare sluka allt i sin omgivning. tack på förhand /elever på ugglumskola
/kalle a, ugglum, göteborg

Svar:
Hej Ugglum! Ni har helt rätt i er invändning!

En kvasar är en extremt ljusstark och avlägsen aktiv galaxkärna. Den överglänser sin värdgalax så mycket, att denna inte tidigare har kunnat observeras. Först med hjälp av CCD-teknik och senare adaptiv optik har många värdgalaxer kunnat påvisas.

Den relativt accepterade teorin är att kvasarer drivs av ett svart hål. De flesta galaxer tycks ha ett svart hål i centrum (se fråga 6228 ). Normalt märks det knappast om rymden omkring hålet är tom.

Om det däremot finns en massa gas och stjärnor, så kommer materialet att dras ner i det svarta hålet. Medan materien accelererar ner i hålet kommer den att kollidera och förvandla rörelseenergi till värme och därmed strålning.

Teoretiskt kan maximalt 50% av totala viloenergin mc2 förvandlas till energi på detta sättet. I fusion av väte till helium förvandlas mindre än 1% av vilomassan till energi. Det är därför svarta hål är en så bra förklaring till de enorma energimängder som strålas ut från kvasarer.

Nedanstående bild från länk 1 visar en kvasar och dess tillhörande galax:

"This image shows quasar PG 0052+251, which is 1.4 thousand million light-years from Earth, at the core of a normal spiral galaxy. Astronomers are surprised to find host galaxies, such as this one, that appear undisturbed by the strong quasar radiation. Quasars reside in a variety of galaxies, from normal to highly disturbed. When seen through ground-based telescopes, these compact, enigmatic light sources resemble stars, yet they are thousand of millions of light-years away and several hundred thousand million times brighter than normal stars. Astronomers believe that a quasar turns on when a massive black hole at the nucleus of a galaxy feeds on gas and stars. As the matter falls into the black hole, intense radiation is emitted. Eventually, the black hole will stop emitting radiation once it consumes all nearby matter. Then it needs debris from a collision of galaxies or another process to provide more fuel.

Credit: John Bahcall (Institute for Advanced Study, Princeton) Mike Disney (University of Wales) and NASA/ESA"



/Peter E

Nyckelord: kvasar [4]; svart hål [51]; galax [28];

1 http://fragelada.fysik.org/index.asp?keyword=svart+h%E5l
2 https://www.spacetelescope.org/images/opo9635a1/

*

Universum-Solen-Planeterna [14135]

Fråga:
Hej! Stora stjärnor avslutar ju sitt liv som en supernova, vad är mekanismen bakom det faktum att dei slutstadiet (när fusionsprocessen skapar järn) imploderar för att sedan explodera, gravitations energin verkar väl inte repulsivt??
/Fredrik A, Brobyskolan, Sunne

Svar:
Hej Fredrik! Nej gravitation är aldrig repulsiv. Exposionen beror på en kombination av två effekter:

Studseffekt: när densiteten blir riktigt hög kommer fallande material att "studsa":

Neutrinos: När stjärnan kollapsar slår sig protoner och elektroner tillsammans och bildar neutroner. I denna reaktion bildas ett enormt flux av neutrinos. Dessa kolliderar med materia utanför centrum och kastar ut en del av stjärnans massa. Resten (centrum) blir kvar som en neutronstjärna eller ett svart hål.

Se mer om supernovor under länken nedan.
/Peter E

1 http://fragelada.fysik.org/index.asp?keyword=supernova

*

Värme [14203]

Fråga:
Hej! Om en isbit läggs i 0,2 kg vatten som är 18 grader, i en termos med värmekapacitet 80 J/K som också är 18 grader, och isbitens massa är 0,05 kg och dess temperatur -15 grader, hur mycket av isen kommer att smälta och varför är det så? Jag får att 40 g smälter, men läroboken får 45 g.
/Annica K, Värmdö, Stockholm

Svar:
Hej Annica! Låt oss först samla lite data om vatten som är nödvändiga för att lösa sådana här problem:

Specifik värmekapacitet för is: 2.1 kJ/kg.K
Smältvärme för is: 333 kJ/kg
Specifik värmekapacitet för vatten: 4,18 kJ/kg.K
Ångbildningsvärme för vatten: 2260 kJ/kg
Avkylning 18->0 grader:
0.2 kg vatten + termosen = 0.2*18*4.18 + 0.080*18 kJ = 15.048 + 1.44 kJ = 16.488 kJ

Uppvärmning -15->0
0.050 kg is = 0.050*15*2.1 kJ = 1.575 kJ

Återstår alltså 16.488 - 1.575 kJ = 14.913 kJ för att smälta x kg is: x*333 = 14.913 kJ, vilket ger x = 14.913/333 = 0.04478 kg = 45 g.

Du har nog glömt att ta hänsyn till värmekapaciteten hos termosen, det är ungefär 10% av vattnets.

Här är data för några vanliga ämnen:
smältvärme - Enthalpy_of_fusion , specifik värmekapacitet - Specific_heat_capacity , ångbildningsvärme - Enthalpy_of_vaporization .

Jämfört med andra ämnen har vatten högt smältvärme, hög specifik värmekapacitet och mycket hög ångbildningsvärme. Detta beror dels på vattnets låga molekylvikt (innehåller många molekyler per kg) och på de vätebindningar som finns mellan närliggande molekyler pga att vattenmolekylen är en dipol. En del av energin går alltså åt för att bryta upp vätebindningar snarare än att "sätta fart" på molekylerna. Water Structure and Science uttrycker det så här:

Water has the highest specific heat of all liquids except ammonia. As water is heated, the increased movement of water causes the hydrogen bonds to bend and break. As the energy absorbed in these processes is not available to increase the kinetic energy of the water, it takes considerable heat to raise water's temperature. Also, as water is a light molecule there are more molecules per gram, than most similar molecules, to absorb this energy. Heat absorbed is given out on cooling, so allowing water to act as a heat reservoir, buffering against changes in temperature.

/Peter E

Nyckelord: vatten/is [49]; specifik värmekapacitet [25];

*

Blandat [14232]

Fråga:
Jag har hört att fysiken är den egentliga naturvetenskapen och all naturvetenskap egentligen är fysik. Är det så?
/Veckans fråga

Ursprunglig fråga:
Jag har hört att fysiken är den egentliga naturvetenskapen och all naturvetenskap egentligen är fysik. Kemi och bilogi skulle alltså vara delar av fysiken. Min lärare säger att det är ur fysiken som kemin kom och ur kemin kom biologin? Vad menas med detta? jag förstår inte riktigt vad min lärare menar när hon säger detta och hon vill inte förklara det mer ingående. mycket tacksam för svar
/Anna L, Gränbyskolan, Uppsala

Svar:
Anna och även Tanja som ställt en liknande fråga!

Att definiera fysik är inte lätt - definitionen beror på sammanhanget. Om man t.ex. menar skolämnet fysik så är det emellertid lätt: fysik är det som ingår i kursplanen för fysikämnet. Om man menar forskningsämnet fysik är det de områden man forskar om på fysiska institutioner vid universiteten. Den preliminära kursplanen för fysik i det nya gymnasiet GY-07 inleds med följande tre stycken:

Ämnet Fysik syftar till fördjupad förståelse för hur fenomen i den egna vardagen och i universum kan förklaras med fysikaliska modeller. Ämnet Fysik syftar också till ett vidgat naturvetenskapligt perspektiv och en modern naturvetenskaplig världsbild. Utbildningen syftar till att öka intresset för ämnet och för fortsatta studier i fysik, matematik, andra naturvetenskapliga ämnen och teknik.

Fysikämnet omfattar allt ifrån det allra minsta, mikrokosmos, till det allra största, makrokosmos. Utbildningen i ämnet syftar till att ge eleven en inblick i olika områden inom fysiken samt dess olika tillämpningar inom vardag, samhälle, industri och forskning.

I all naturvetenskap sker utveckling i samspel mellan experiment och teori. Teorier och modeller är mänskliga tankekonstruktioner som ständigt utvecklas och påverkar människans världsbild. Utbildningen i Fysik syftar därför till ökad kunskap om fysikens arbetsmetoder samt om hur fysikens kunskapsområden utvecklas i samspelet mellan fysik, andra naturvetenskapliga ämnen och matematik.

En alternativ definition är: fysik är den vetenskap som beskriver materia, energi och krafter. Då fysiken är en vetenskap tillämpas den vetenskapliga metoden (se vetenskaplig metod ), med uppställande av hypoteser som antingen förkastas eller antas på grundval av experiment och observationer.

Man skiljer även på klassisk fysik (allt före 1900, t.ex. mekanik, elektromagnetism) och modern fysik (t.ex. relativitetsteori, kvantmekanik, elementarpartikelfysik). Sedan har man även grundläggande fysik (forskning inom fysik som motiveras av vår vilja att förstå naturen) och tillämpad fysik (tekniska, medicinska, mm tillämpningar av fysik).

Eftersom modern fysik omfattar vetande från i stort sett hela 1900-talet, kan det vara på sin plats att använda begreppet nutida fysik som skulle innefatta partikelfysik (standardmodellen ), kosmologi (big bang ), plasmafysik och fusion (fusion ) och kärnfysik, se länk 1 för detaljer om Contemporary Physics Education Project (CPEP).

Historiskt var det så att fysik var all naturvetenskap. Efter hand som kunskapen ökade, hade man behov av specialisering. Efter hand frigjordes biologi, geologi, kemi, m.fl. och blev egna ämnen. Vad din lärare antagligen menar är att t.ex. biologin bestäms helt och hållet av arvet genom DNA-molekylen. De lagar som styr DNAs egenskaper är fysik (kvantmekanik). DNA-molekylernas egenskaper är emellertid så komplicerade att vi inte kommer särskilt långt i förståelsen med kvantmekanik. Man måsta använda andra metoder för att komma framåt, och då är det praktiskt att ge ämnet en egen beteckning.

Historiskt har biologin utvecklats helt oberoende av fysiken: man studerade och klassificerade organismer (Linné, 1700-talet), och efter hand förstod man arv och utveckling (Mendel, Darwin, 1800-talet) och DNA upptäcktes (Watson, Crick, Wilkins och Rosalind Franklin, 1950-talet, se länk 2). Det var egentligen först då kopplingen till fysik återuppstod i form av t.ex. biofysik. Se vidare fråga 13720 .

Även det vi i dag kallar fysik har delats upp i separata ämnen: i början av 1900-talet kallades allt som hade att göra med atomer för atomfysik. Efter hand som kunskaperna ökade frigjordes kärnfysik (som beskrev atomkärnan, atomfysik beskrev bara elektronerna kring kärnan). Efter 1950 frigjordes sedan elementarpartikelfysiken från kärnfysik och blev ett eget ämne som behandlar elementarpartiklar och deras växelverkningar.

För att visa hur krånglig och godtycklig uppdelningen är kan jag säga att delar av kärnfysik i USA klassificeras som kemi (Nuclear Chemistry).

Mer om vad fysik är och vad det är bra för finns under nedanstående länkar och här: fysik, nytta med . Jag beklagar om svaret inte är lätt att förstå, det går nog tyvärr inte att förklara det enklare.

Texten på tröjan nedan antyder att fysiken är den ultimata vetenskapen .



/Peter E

Nyckelord: fysik [10]; *biologi [20]; vetenskaplig metod [18]; kursplan [3];

1 http://www.cpepweb.org/
2 http://www.chemheritage.org/discover/online-resources/chemistry-in-history/themes/biomolecules/dna/watson-crick-wilkins-franklin.aspx

*

Blandat, Energi, Materiens innersta-Atomer-Kärnor [14237]

Fråga:
Randell Mills och pseudovetenskap
/Veckans fråga

Ursprunglig fråga:
En amerikan som heter Randell Mills har tydligen uppfunnit ett system för att få billig och miljövänlig energi från väte. Jag skulle vilja veta mer om detta!
/Sven E, Stockholm

Svar:
Det du frågar om är nog det mest avancerade exemplet på pseudovetenskap och bluff och båg i hela vetenskapshistorien! Nationalencyklopedin säger om pseudovetenskap: mystisk eller spekulativ forskning som inte är accepterad av vetenskapssamhället, t.ex. alkemi, parapsykologi och astrologi. Fallet Mills påminner en hel del om Kall fusion (se fråga 2409 ), men det är ett mycket mer avancerat bedrägeri.

Länken HYDROGEN IS POTENTIAL NEW ENERGY SOURCE beskriver vad idén är. I korthet går det ut på att kvantmekaniken är fel och att det tillstånd vi kallar grundtillståndet i väte inte är det lägsta tillståndet. Det finns enligt Mills flera mycket lägre liggande tillstånd, och man kan genom katalys med kalium få elektronen att gå till dessa lägre tillstånd. Väte i de lägre tillstånden kallar han hydrino. Man skulle då kunna utvinna c:a 40 till flera hundra eV energi från varje atom. Energiutvecklingen ger upphov till en plasma, vilket med vad som kallas en gyrotron kan transformeras till mikrovågor som i sin tur kan generera elektricitet.

Det är uppenbart att om allt detta vore sant skulle man t.ex. kunna köra bilar med vatten som bränsle! Uppfinningen skulle representera ett enormt ekonomiskt och miljömässigt värde. Mills har bildat ett bolag med en mycket professionell webbsajt BlackLight Power, Inc. , och investerare har satsat mycket pengar. Bolaget säljer rättigheter till Mills uppfinning. Bolaget har en fin anläggning i New Jersey, men det tycks bara bestå av direktörer .

Blacklight_Power är en balanserad sammanfattning av Mills idéer och patent. Det är ingen överdrift att säga att etablerade fysiker är måttligt imponerade, och de som är positiva tycks vara direktörer i Mills bolag.

Mills har även skrivit en bok The Grand Unified Theory of Classical Quantum Mechanics med helt nya teorier vad gäller fysiken. Boken (på 1800 sidor i 3 volymer!) ser mycket vederhäftig ut och måste vara resultatet av mycket arbete. Mycket i boken är korrekta textbokskunskaper och en del är fullständig rappakalja. Bland detta finns helt nya vinklingar och teorier, bland annat de nya lägre liggande tillstånden i väte.

En annan intressant sak är att Mills "härleder" förhållanden mellan elementarpartiklarnas massor som funktion av finstrukturkonstanten (en dimensionslös konstant med värdet 1/137.03599911 som förekommer i elektromagnetiska teorin). Bara detta, om det vore korrekt, skulle ge nobelpriset direkt eftersom Standardmodellen för elementarpartiklar och kraftverkningar inte ger några värden på dessa.

____________________

I detta sammanhang kan det vara på sin plats att diskutera vad naturvetenskap är och vad som å andra sidan är pseudovetenskap.

Naturvetenskap - vetenskaplig metod

Ett naturvetenskapligt arbetssätt är ett ständigt samspel mellan teoribyggande och observationer:

TEORI/MODELL <<--->> OBSERVATIONER/EXPERIMENT

I övrigt är följande punkter viktiga för vetenskaplig metod:

  • Alla resutat skall rapporteras i öppna tidskrifter
  • Resultat skall vara testbara och reproducerbara
  • Man får inte godtyckligt välja resultat som "passar"
  • Enklast möjliga beskrivning som inte strider mot tidigare observationer föredras
  • Acceptans för nya idéer – villighet att ompröva gamla teorier
  • Sakargument, ej status och "det är skrivet", bestämmer trovärdighet

Se även ett par frågor som behandlar detta under vetenskaplig metod .

Vad är inte vetenskap?

Vetenskap innefattar alltså det som är mätbart och testbart. Resultat av experiment skall vara reproducerbara. Pseudovetenskap å andra sidan karakteriseras av

  • Data är ofta anekdotiska
    - En vän jag litar på sa att han såg varelser stiga ut ur UFOt
  • Uttalanden är ofta mycket kategoriska eller mycket vaga
    - På vetenskapens nuvarande stadium kan vi säga att...
  • Brist på andra förklaringar
    - Att vi inte kan "förklara" ett fenomen betyder inte att det är övernaturligt
  • Slumpmässiga sammanträffanden är möjliga
    - Bara för att två händelser sker samtidigt behöver inte betyda att de är beroende av varandra
  • Referenser är vaga - auktoriteter används ofta
    - Den välkände Professor Bloggs vid CalTech säger att... I vetenskapen ger man en referens som kan kontrolleras. Det är en persons arbete (normalt publikation) som skall bedömas, inte personens tillförlitlighet
  • Vetenskapligt språk används ofta
    - Bitvis kan fakta vara korrekta – men de är ofta triviala eller irellevanta fakta från läroböcker
  • Hänvisningar till religiösa skrifter
    - Saknar bevisvärde – anektotiska och innehåller ofta symbolik som inte får övertolkas

Observera att det är skillnad på pseudovetenskap, där villfarelsen är avsiktlig, och dålig vetenskap, som i bästa fall kan vara ett oavsiktligt misstag.

Skepticism/källkritik i webbsökningar

Det finns t.ex. på internet väldigt mycket bra information, men också mycket skräp och pseudovetenskap. Några tips för att bedöma information:

  • Leta efter oberoende bekräftelse av fakta
  • Var öppen för olika åsikter, men använd bara bevis som kan bekräftas
  • Studera olika hypoteser – skaffa dig inte en favoritteori som utesluter alla andra
  • Kvantifiera där det är möjligt - "vad som är vagt och kvalitativt är öppet till många förklaringar"
  • Använd "Occam’s Razor": börja med den enklaste förklaringen
  • Är ett uttalande öppet för experimentell bekräftelse? Om inte, kan det vara intressant att diskutera, men validiteten kan aldrig bekräftas
  • Är urspungspersonen knuten till en reputabel institution? Reputabel institution är dock varken nödvändigt eller tillräckligt villkor för tillförlitlighet! (Einstein var anställd vid en patentbyrå i Bern när han pubicerade sina första papper.)
  • Ligger sidan under en officiell website från en respektabel institution (renomerat universitet eller forskningsinstitution)
  • Är sidan från en publikation i en renomerad tidskrift så kan man oftast lita på uppgifterna
  • Icke granskade open access system (t.ex. arXiv, länk 2) kan innehålla allt från nobelpris-forskning till pseudovetenskap, så här gäller det att vara försiktig
  • Är ursprungspersonen expert på ämnet? Nobelpristagare och professorer missbrukar tyvärr ibland sin status för uttalanden i helt andra ämnen!
  • Var misstänksam om författaren utan mycket starka argument angriper grundläggande, sedan länge etablerad vetenskap

Uppslagsverk på webben

Wikipedia är ett flerspråkigt webbaserat uppslagsverk med i huvudsak fritt och öppet innehåll som utvecklas av sina användare (ofta benämnda wikipedianer).

Wikipedia har mer och mer blivit en standardkälla för information. De svenska versionen är ganska begränsad, och jag rekommenderar den bara för svenska förhållanden. Den engelska versionen är emellertid mycket omfattande. Wikipedia kritiseras ibland för att den skulle vara otillförlitlig eftersom vem som helst kan skriva artiklar. Det är ju precis detta som är Wikipedias styrka! Om något fel kommer in så rättas den snabbt av någon annan. Om det är oenighet i ett ämne så markeras detta ofta tydligt i artikeln. Wikipedia innehåller faktiskt inte mycket fler fel och saknad information än Nationalencyklopedin (NE) enligt en undersökning man gjort på Sveriges Radio. Wikipedia är naturligtvis inte den slutliga källan när det gäller komlicerade begrepp, men den är en utmärkt utgångspunkt.

Nationalencyklopedin (NE, Nationalencyklopedin ) har fördelen att artiklarna skrivits av experter på det aktuella området, så kvalitén är hög och jämn. Nackdelen är att nyheter kommer in mycket långsammare än i Wikipedia där "nördar" (positivt menat) bevakar allt som händer. Wikipedias styrka är initierade artiklar även i ganska udda ämnen. Den pedagogiska nivån är emellertid mycket varierande. Avvikelser från Wikipedias principer markeras emellertid oftast tydligt.

En annan fördel med Wikipedia jämfört med NE är att alla viktiga fakta skall ha en referens till en originalkälla. Om detta inte är fallet för artiklar med lägre kvalité så signaleras detta oftast längst upp på sidan.

En stor fördel med Wikipedia är att många bilder är helt fria under Wikimedia Commons .

Pseudovetenskap mm

Ett vanligt pseudovetenskapligt trick är att man påstår något som inte har något stöd i teori eller experiment, men som å andra sidan inte kan motbevisas. Detta kallas efter filosofen Bertrand Russell för Russells tekanna.

Russells tekanna eller den himmelska tekannan är en analogi av filosofen Bertrand Russell. Analogin är ett argument mot idén att det är en skeptikers uppgift att motbevisa religiösa dogmer, snarare än den troendes uppgift att bevisa dem, se Russells_tekanna .

Länk 1 innehåller artiklar om och exempel på pseudovetenskap. Nature of Science är en utmärkt interaktiv site om vetenskaplig metod. James Randi Educational Foundation är hemsidan för en av de mest kända förkämparna för vetenskaplig metod och mot pseudovetenskap. Wikipedia-artiklarna Vetenskap och Pseudovetenskap är mycket bra. Se även Russell's_teapot , Science , Scientific_method , Pathological_science och Pseudoscience .

Referenser: bland annat Carl Sagan: Demon-Hunted World, Robert L. Park: Voodoo Science: The Road from Foolishness to Fraud och Bennett, Shostak, Jakosky: Life in the Universe.

Se även nedanstående figur (Image credit: Hemant Mehta of the Friendly Atheist blog).



/Peter E

Nyckelord: pseudovetenskap [11]; vetenskaplig metod [18]; Wikipedia [5];

1 http://fragelada.fysik.org/links/search.asp?keyword=pseudovetenskap
2 http://arxiv.org/

*

Universum-Solen-Planeterna [14367]

Fråga:
Energi från ett svart hål.
/Veckans fråga

Ursprunglig fråga:
Tja, jag läste att när en massiv stjärna dör och övergår till ett svarthål en s.k. supernova så kan energi skickas ut i form av gravitations vågor. ibland kan det röra sig om så mkt som 10^44 joule. Vart kommer denna energi ifrån är den totala massan mindre efter?
/Karl J, Hjärteskolan, Trosa

Svar:
Energin kommer från bindningenergin. En massa som faller ner i ett svart hål binds av gravitationsfältet på samma sätt som en elektron binds i en atom. Elektronen skickar ut ljus när den övergår till lägre tillstånd. Det kan även infallande materia i ett svart hål göra genom kollisioner och uppvärmning, men en massa som rör sig snabbt kan även sända ut gravitationsvågor .

Den energi som sänds ut som elektromagnetisk strålning eller gravitationsvågor är förlorad, så massan av det kompakta objektet minskar med detta belopp. Låt oss titta lite närmare på energiförhållandena.

Klassiskt (Newton) är flykthastigheten från en massa M med radien R är lika med ljushastigheten c när

R = RS = 2GM/c2

(flykthastigheten är v = (2GM/r)1/2, se fråga 3782 ). Gravitationell bindningsenergi för en massa m vid ytan (kallas händelsehorisonten eller Schwarzschild-radien) av ett svart hål blir då

GMm/RS = mc2/2

vilket är exakt halva vilomassan mc2. Om man i stället använder den allmänna relativitetsteorin (vilket vi självklart måste göra) blir uttrycket för händelsehorisonten oförändrad men den gravitationella bindningsenergin blir lika med vilomassan mc2.

Hur skall vi tolka detta? Om vi låter en massa m falla ner i ett svart hål kan vi frigöra maximalt energin mc2/2. Resten kommer att försvinna som rödförskjutning. Ett svart hål är alltså en mycket effektiv energikälla - fusion frigör t.ex. bara någon procent av vilomassan. Detta är orsaken till att man tror att de mest energetiska objekten vi känner till, t.ex. kvasarer, är svarta hål. Om energin frigöres när massan är vid händelsehorisonten blir rödskiftet oändligt, och ingen energi slipper ut. Om vi emellertid låter energin stråla ut när massan är på väg ner, så kan en del av energin slippa ut - maximalt mc2/2.

Länk 1 innehåller information från en expert på området. Länk 2 är en användbar formelsamling för svarta hål. Se även Black_hole och Supermassive_black_hole .
/Peter E

Nyckelord: svart hål [51]; gravitationsvågor [19]; flykthastighet [4]; relativitetsteorin, allmänna [33];

1 http://www2.physics.umd.edu/~tajac/
2 http://fragelada.fysik.org/resurser/blackholes.pdf

*

Energi, Materiens innersta-Atomer-Kärnor [14443]

Fråga:
Vad är densiteten hos plasmat i fusion?
/Veckans fråga

Ursprunglig fråga:
Hej, jag undrar vad densiteten hos plasmat i fusion är?
/Niklas F

Svar:
Det intressanta är inte densititen utan kombinationen densitet och inneslutningstid. Det är då det heta plasmat har möjlighet att producera tillräckligt många fusionsreaktioner. Som framgår av länk 1 fordras för deuterium-tritium fusion att

   nt >= 1014 s/cm3

där n är densiteten i partiklar/cm3 och t är inneslutningstiden i sekunder. För en rimlig inneslutningstid på 1 sekund blir alltså densiteten 1014 /cm3. Detta kan jämföras med densititen för luft vid normalt tryck och temperatur c:a 3*1019 /cm3. Densiteten för plasmat är alltså ett hyggligt förvakuum på c:a 2*10-3 torr!
/Peter E

Nyckelord: fusion [17];

1 http://hyperphysics.phy-astr.gsu.edu/hbase/nucene/lawson.html#c4

*

Universum-Solen-Planeterna [14493]

Fråga:
Frågor om stjärnors utveckling
/Veckans fråga

Ursprunglig fråga:
Hej! Jag har några frågor angående ett skolarbete. Skulle vara tacksam för svar.

1. Man vet att stjärnor föds ur stora nebulosorna, men hur vet man det?

2. När en stjärna som är ungefär i solens storlek har blivit en rödjätte, vad händer efter det?

3. Stjärnor består ju av väte som via fusion omvandlas till helium, hur vet man det här?

4. En stjärna har ju en speciel livscykel, hur vet man just att denna livscykel är som den är. Hur vet man t ex att stjärnorna övergår i olika faser?

5. Hur kommer det sig att vissa stjärnor får solsystem och vissa inte? Hur vet vi det?
/Cecilia B, Thorildsplansgymnasium, Stockholm

Svar:
Hej MsPluto!

1. Man kan observera stjärnor på alla stadier av utveckling, inklusive stjärnor som håller på att bildas, se stjärnors utveckling och solsystemets bildande .

2. En vit dvärg är en normalstor stjärna som kollapsat till en mycket liten storlek efter att den gjort slut på sitt kärnbränsle. En typisk vit dvärg har en radie som är 1 procent av solens, men den har grovt räknat samma massa.

När vätet i centrum tar slut fortsätter förbränningen i ett skal runt stjärnan. Stjärnan blir då en röd jättestjärna. När bränslet i skalet tar slut komprimeras kärnan ytterligare och förbränning av helium till kol börjar. Mot slutet av denna process expanderar yttre delen av stjärnan och bildar en planetarisk nebulosa, kärnan blir en vit dvärg. Bilden nedan visar ett sådant system som kallas 'Cats Eye nebula'. Se vidare White_dwarf#Formation , länk 1 och 2.

3. Kunskapen om fusionsprocessen är mycket god. Antagandet förklarar alla observationer, bl.a. att heliumförekomsten är högre är de 24% som fanns efter big bang i gas som funnits i en stjärna.

4. Genom observationer av stjärnor av olika typ samt modellberäkningar av utvecklingen. Detta senare är mycket viktigt.

5. Sjärnor som har mycket av tunga grundämnen (nya stjärnor) har mycket oftare planetsystem än stjärnor som innehåller lite av tunga grundämnen. Med tunga grundämnen menar vi ämnen tyngre än helium. En gas bestående av enbart väte och helium kan inte bilda planeter - det behövs även ämnen som kondenseras ("klibbar"). Vatten (H2O) är säkert viktigt dels beroende på dess egenskaper och dels eftersom syre är det ämne det finns mest av med undantag för väte och helium. Se vidare exoplaneter .

Mer om stjärnutveckling: Stellar_evolution .



/Peter E

Nyckelord: stjärnors utveckling [15]; nebulosa [13];

1 http://imagine.gsfc.nasa.gov/docs/science/know_l1/dwarfs.html
2 http://astronomy.nmsu.edu/geas/lectures/lecture24/slide03.html

*

Energi [14598]

Fråga:
Angående 14593 och 814. Ni skriver att vågkraft är förnyelsebar men att tidvattenkraft inte är det. Tidvattenkraften upphör ju att fungera när jorden och månen kommer för långt ifrån varandra, medans vågkraften upphör när solen slocknar. Ser man det så borde ju bägge vara icke-förnyelsebara. Finns det någon 'vedertagen' definition av förnyelsebar/ej förnyelsebar?
/Jonas A, Bäckaskolan, Umeå

Svar:
Jonas! Det finns såvitt jag vet ingen sträng definition vad gäller tiskonstanten för vad som kan kallas förnyelsebart. Jag säger också att effekten på jordens rotation är liten, vilket betyder att i praktiken är tidvattenenergi förnyelsebar. Det vore absurt att inte kalla solenergi förnyelsebar, eftersom utan solen finns vi inte och då behöver vi ingen energi.

Det viktiga är inte förnyelsebarheten utan det potentiella innehållet i en källa. Fusionen av väte i solen har en enorm potential som energikälla. Det är detta som är det viktiga, inte förnyelsebarheten - är källan i stort sett oändlig är ju förnyelsebarheten irrelevant.

Jag skulle sätta gränsen på tidskonstant om potentialen är liten vid några tiotal år. Torv med en möjlig förnyelsetid på c:a 1000 år är alltså inte förnyelsebar.
/Peter E

Se även fråga 814

*

Energi [14753]

Fråga:
Hej, Jag håller på att göra en fördjupning inom Fusion som framtida energikälla. Jag skulle uppskatta om ni kunde ge eran expertis och tankar om några frågor jag har. Tack på förhand.

Tror ni Fusion kommer bli framtidens Energikälla?

Hur lång tid tror ni isåfall det kommer ta innan det kan börja användas som Energikälla?

För- och Nackdelar?

Finns det bättre alternativ?

Övrigt?

Anledningen till att jag vill få reda på era synpunkter om ämnet är för att jag vill bilda en sån bred bild om ämnet som möjligt!
/Fredrik R, Värmdö Distans, Wien

Svar:
Nej, jag tror inte på fusion som framtidens energikälla. På 1960-talet sas det att fusionsenergin skulle vara användbar inom 30 år. I dag säger man 50 år, kanske. Inte särskilt övertygande! Man sa att JET i England skulle vara den sista testreaktorn och att ITER skulle visa hur man fick ut energin. Nu planerar man göra detta först i nästa (ännu dyrare) generation Tokamak.

Fördelen med fusion som energikälla är att det finns nästan obegränsat med bränsle: deuterium och tritium (som tillverkas från litium). Fusion ger heller inget (nåja, lite) radioaktivt avfall. Nackdelen är att man inte vet om det fungerar. Jag tror emellertid att transmutation (se nedan) är mer lovande.

Fusion går till så att man innesluter ett mycket hett plasma av deuterium och tritium i ett magnetfält. Kärnreaktioner äger rum och energi genereras (se bilden), men svårigheten är att ta vara på energin. Eftersom laddade partiklar inte kan ta sig ut genom magnetfältet, så får man använda sig av neutronerna som bildas. Dessa har hög energi, men man vet ännu inte vilket material man kan använda som dels på ett effektivt sätt tar ut energin från neutronerna och dels klarar ett enormt bombardemang av neutroner.

Se vidare fusion , energikällor och transmutation av kärnavfall samt nedanstående länkar.



/Peter E

Nyckelord: fusion [17];

1 http://en.wikipedia.org/wiki/Nuclear_fusion
2 http://en.wikipedia.org/wiki/Fusion_power

*

Universum-Solen-Planeterna, Värme [14839]

Fråga:
Jag har hört att det tar en miljon år för en ljusstråle att färdas från solens centrum till ytan. Detta ska tydligen bero på solens höga densitet. Är detta rimligt?
/Johannes R, Brogårdsgymnasiet, Kristinehamn

Svar:
Hej Johannes! Ja, det låter otroligt men det är sant att det tar mycket lång tid. Det beror inte på att solens densitet är särskilt hög utan det beror på att opaciteten (ett mått på ogenomskinligheten) är stor. Vad som händer är att fotonerna diffunderar ut från centrum med vad som kallas slumppromenad (random walk). Fotonerna absorberas och återutsänds i en slumpmässig riktning hela tiden, så den effektiva hastigheten ut mot solytan blir mycket liten. Se vidare diffusion och länk 1.
/Peter E

1 http://www.opencourse.info/astronomy/introduction/12.sun_interior/

*

Materiens innersta-Atomer-Kärnor [14847]

Fråga:
Vad är skillnaden mellan fission och fusion?
/Veckans fråga

Ursprunglig fråga:
Jag undrar hur jag enkelt förklarar skillnaden mellan fusion och fission för mina mellanstadieelever. Jag fick frågan av en elev som är väldigt intresserad av svaret men resten av klassen kommer sitta som fågelholkar när jag förklarar. Undrar därför om det finns en enkel, kort och koncis förklaring... Jag vill ju att de också ska förstå lite...
/Annica W, Centralskolan, Åtvidaberg

Svar:
Annica! Det är inte så lätt att förklara med mellanstadieelevernas begreppsbild. För full förståelse behöver man t.ex. förstå ett begrepp som bindningsenergi.

Det enklaste svaret är bara beskrivande: fusion är när man slår ihop lätta kärnor och fission är när man klyver tunga kärnor. Båda dessa processer ger energi (värme), så de kan användas t.ex. för att producera elektricitet.

Fissionsenergi är väl etablerat i praktiken i kärnkraftverk. Man klyver urankärnor genom att bombardera dem med neutroner. Eftersom det produceras 2-3 neutroner i varje fissionsprocess, går det att åstadkomma en kedjereaktion som kan underhållas kontinuerligt.

Fusionsenergi är däremot än så länge bara ett framtidshopp. Som en illustration till svårigheterna kan jag berätta att när jag började studera kärnfysik för drygt 30 år sedan så sade man att det kommer att ta 30 år att realisera en energiproducerande fusionsreaktor. I dag är uppskattningen: kanske om 50 år! Detta visar om inget annat hur svårt problemet är.

Anledningen till att kontrollerad fusion är så svår är att man försöker slå ihop två atomkärnor som är positivt laddade. Lika laddningar repellerar varandra, så för att kärnorna skall komma tillräckligt nära varandra så måste de skjutas mot varandra med hög hastighet. Hög hastighet hos atomerna i en gas betyder hög temperatur - flera miljoner grader. Man behöver kunna hålla ihop gasen och hindra den att expandera. Detta kan man göra med magnetfält, men det återstår ännu många problem att lösa. Nästa generation av försöksanläggning ITER, som är ett globalt samarbetsprojekt, håller på att byggas i Frankrike, se ITER .

Fusion sker i alla stjärnor, inklusive solen, så solenergi och vindenergi är i princip fusionsenergi från en naturlig fusionsreaktor i solens centrum.

Låt oss se om vi kan förstå varför man kan utvinna energi både genom att slå samman lätta kärnor och att klyva tunga kärnor.

Atomkärnan består av positivt laddade protoner (vätekärnor) och neutrala neutroner. Protoner och neutroner kallas med ett gemensamt namn för nukleoner. Antalet nukleoner kallas masstal och betecknas med A. Antal protoner i en kärna kallas atomnummer och betecknas med Z. Det är atomnumret som bestämmer vilket grundämne man har att göra med.

Protonerna repellerar visserligen varandra, men det finns attraherande krafter mellan nukleonerna som är starkare är repulsionen. Nukleonerna kommer därför att bindas samman och ha vad vi kallar en bindningsenergi.

Man kan förstå förvånansvärt mycket av atomkärnors egenskaper genom en mycket enkel modell: vätskedroppsmodellen. Man betraktar atomkärnan som en vätskedroppe - t.ex. en vattendroppe - så att nukleonerna motsvarar vattenmolekyler. Vattenmolekylerna i en vattendroppe binds samman genom krafter mellan närliggande molekyler, dvs den attraktiva kraften har kort räckvidd. Molekylerna i en vattendroppe har också en bindningsenergi - man måste tillföra energi för att "koka bort" molekyler. Se Semi-empirical_mass_formula för mer om vätskedroppsmodellen.

Bindningsenergin per nukleon visas i nedanstående figur. Grovt kan man säga att bindningsenergin för de flesta kärnor är c:a 8-9 MeV per nukleon. För lätta kärnor är bindningsenergin lägre, och den minskar även för mycket tunga kärnor. De mest stabila kärnorna - högst bindningsenergi - finns omkring masstalet 60, dvs järn och nickel.

Den lägre bindningsenergin för lätta kärnor förklaras av att små kärnor har relativt mycket "yta". Nukleonerna på ytan har inga grannar "utåt", så bindningen blir mindre. Det är denna effekt som orsakar ytspänning i en vattendroppe, se ytspänning .

Nedgången i bindningsenergi för tunga kärnor beror på repulsionen mellan protonerna. Coulomb-repulsionen har lång räckvidd till skillnad från attraktionskraften mellan nukleonerna som har kort räckvidd. Detta betyder att bindningen går som masstalet A och repulsionen som Z(Z-1)/2 där Z är kärnladdningen (antal protoner). För kärnor med många protoner kommer därför coulomb-repulsionen att bli större och därmed bindningsenergin att minska.

Låt oss titta lite på energiförhållandena för fission och fusion.

Om vi delar en urankärna med A c:a 240 hamnar vi omkring A=120. Bindningsenergin per nukleon är 7.5 vid A=240 och 8.4 vid A=120 (se figuren nedan). Vi vinner alltså en bindningsenergi på c:a (8.4-7.5)*240=216 MeV. Detta är ett mycket högt värde för en kärnreaktion, och är anledningen till att det går att utvinna så mycket energi genom fission av tunga kärnor.

Kvalitativt kan man även förstå fissionsprocessen med vätskedroppsmodellen: en inkommande neutron sätter urankärnan i svängning. Om deformationen har tillräckligt stor amplitud, kommer coulomb-repulsionen att ta överhanden och kärnan kan delas i två delar.

Den mest effektiva fusionsreaktionen är att slå ihop deuterium med tritium:

d + 3H --> 4He + n

Bindningsenergierna (Nuclear_binding_energy ) för de ingående kärnorna är enligt figuren nedan

2H: 2*1.1=2.2 MeV
3H: 3*2.8=8.4 MeV
4He: 4*7.0=28.0 Mev
n: 0 MeV

Differensen i bindningsenergi blir alltså 28.0-(2.2+8.4)=17.4 MeV. Som synes är anledningen till den stora frigjorda energin att 4He-kärnan (alfapartikeln) är mycket stabil. Detta är det enda man inte kan förstå med den enkla vätskedroppsmodellen - för att förstå detta behöver man kvantmekanik.

Vätskedroppsmodellen kan även förklara vilken kärna för ett giver masstal är stabilast, se Semi-empirical_mass_formula#Examples_for_consequences_of_the_formula . Även massparablerna (fråga 13758 ) förklaras bra av massformeln.

Hoppas du kan använda en något av ovanstående utan alltför mycket fågelholksreaktion. Mer om ämnet finns under nedanstående länkar (på engelska): länk 1 är mer om bindningsenergi och länk 2 om kärnenergi.



/Peter E

Nyckelord: bindningsenergi [23]; fusion [17]; fission [15]; kärnenergi [19]; vätskedroppsmodellen [5]; kärnreaktion [5];

1 http://hyperphysics.phy-astr.gsu.edu/hbase/nucene/nucbin.html
2 http://science.howstuffworks.com/nuclear-power.htm

*

Materiens innersta-Atomer-Kärnor [14997]

Fråga:
Hej! Inkluderar Coulombs barriär kärnans radie, eller varifrån gäller avståndet? Om Coulombs barriär skulle gälla f.r.o.m. kärnans centrum, borde inte det innebära att ju större kärnor desto lättare att bryta Coulombs barriär (som ex. vid fusion o fission)? Är det därför som väte har så stark bindning?
/Lisen T, Kärrtorps gymnasium, Enskede

Svar:
Lisen! Eftersom laddningstätheten varierar som funktion av avståndet och det inte finns någon skarp gränslinje, så är radien hos en atomkärna dåligt definierad. Radien beror på hur man bestämmer den. En vanlig definition är att radien för en kärna är där man ser inflytande från den attraktiva kärnkraften: om man sprider alfa-partiklar med ökande energi mot en kärna följer spridningen först en kurva som ges av coulomb-kraften. Vid en viss energi ser man en avvikelse och den motsvarande radie blir kärnans radie. Se vidare länk 1.

En annan metod att bestämma radien är att bestämma laddningstätheten som funktion avståndet från centrum genom elektronspridning. Man definierar sedan kärnradien där laddningstätheten har halva värdet av tätheten i centrum. Se länk 2.
/Peter E

Nyckelord: kärndata [3];

1 http://hyperphysics.phy-astr.gsu.edu/Hbase/nuclear/rutsca3.html#c4
2 http://hyperphysics.phy-astr.gsu.edu/Hbase/nuclear/nucuni.html#c4

*

Universum-Solen-Planeterna [15088]

Fråga:
Hej jag undrar ifall solens densitet blir större eller mindre i mitten av solen jämfört med desiteten på solens kant? eftersom det här 16 miljoner C men också väldigt hög dragnings kraft.
/david s

Svar:
Hej David! Solens densitet ökar kraftigt från ytan som vi ser (fotosfären) med en densitet som är mycket lägre än jordens atmosfär till centrum där densiteten är 162200 kg/m3. Medeldensiteten är 1408 kg/m3, vilket är lite mer än vattens densitet.

Temperaturen är knappt 6000 K på ytan, medan den i centrum, där energiproduktionen (fusion av väte till helium) sker, är 15 miljoner K.
/Peter E

Nyckelord: solens energiproduktion [9];

1 http://nssdc.gsfc.nasa.gov/planetary/factsheet/sunfact.html
2 http://en.wikipedia.org/wiki/Sun

*

Universum-Solen-Planeterna [15237]

Fråga:
Hej! Universum består av mörk materia, mörk energi, strålning och materia och på något frågesvar finns procentsatserna. Kommer procentsatserna att vara konstanta framdeles eller, om de ändras, hur går utvecklingen i det långa loppet? Även om vi kommer på vad mörk materia är så gäller frågan om dess procenttal ändras.
/Thomas Å, Arlandagymnasiet, Märsta

Svar:
Hej Thomas! Typisk fråga som ej går att svara bestämt på men man kan åtminstone ha lite funderingar

Frågan som ger universums sammansättning är nummer 11568, se nedanstående figur. Eftersom man inte vet vad mörk energi och mörk materia har för egenskaper, kan man inte säga något om ändring i sammansättningen vad gäller dessa. Vanlig materia (innefattande strålning) ändrar emellertid sammansättning lite grann genom att väte förvandlas till helium i stjärnorna.

Bindningsenergin för 4He är 28 MeV, så fusionsprocessen förvandlar 28/(4*1000) = 0.007 = 0.7% av massan till energi (strålning). Med tanke på att inte allt väte konsumeras så är detta en liten effekt.

En större effekt kan vara att skapandet av svarta hål kan ge upphov till strålning - maximalt mc2/2, se fråga 14367.

Det finns mycket intressant information i Ask an Astronomer under Cosmology And The Big Bang.



/Peter E

Se även fråga 11568 och fråga 14367

Nyckelord: big bang [37]; mörk materia [17];

*

Universum-Solen-Planeterna [15274]

Fråga:
Föds det fler stjärnor än det dör i universum? Kommer det att innebära slutet för universum om det dör fler stjärnor än det föds?
/Simon Y, Ebersteinska gymnasiet, Norrköping

Svar:
Simon! Låt oss först definiera vad en stjärna är: det är en himlakropp bestående av gas (normalt mest väte och helium) i joniserad form (plasma) som producerar energi genom fusion. Denna energi strålar ut i form av ljus. Även vita dvärgstjärnor och neutronstjärnor kallas lite oegentligt för stjärnor, men de är egentligen stjärnrester som inte producerar någon energi.

Eftersom det krävs en temperatur på minst några miljoner K för att fusion skall komma igång, så måste en stjärna ha en massa på minst 0.08 solmassor. Annars är den vad man kallar en brun dvärg.

Det är svårt att säga om det föds fler stjärnor än det dör. Dels varierar livslängden mycket (massiva stjärnor lever kort tid, lätta mycket länge) och dels sker bildandet av stjärnor på vissa ställen där det finns gas, medan det på andra ställen inte bildas några stjärnor. Stjärnorna kommer i varje fall inte att ta "slut" ens när solen slocknar vilket är om drygt 5 miljarder år.

Se Stjärna och Star för mer om stjärnor.
/Peter E

Nyckelord: stjärnors utveckling [15];

*

Universum-Solen-Planeterna [15342]

Fråga:
Varför är livslängden för massiva stjärnor mycket mindre är för lätta?
/Veckans fråga

Ursprunglig fråga:
I vår lärobok finns det en tabell över stjärnors livslängd i förhållande till deras storlek. 0,1 solmassor = 100 miljarder år, 1 solmassa = 10 miljarder år, 10 solmassor = 10 miljoner år, 100 solmassor = 1 miljon år. Antalet solmassor blir 10 gånger större för varje steg itabellen, borde då inte livslängden bli 10 gånger mindre för varje steg? Står det fel i boken eller är det så att stjärnor med 1 solmassa lever 1000 gånger så länge som stjärnor med 10 solmassor?
/Teodor N, Västbergaskolan, Stockholm

Svar:
Genom att bestämma massan, avståndet och ljusstyrkan för ett stort antal stjärnor kan man se att det finns en samband mellan massa och ljusstyrka för stjärnor som befinner på huvudserien, se figuren nedan från länk 1. Eftersom sambandet i log-log plotten kan approximeras med en rät linje kan vi bestämma följande approximativa samband (mass-luminositetsrelationen):

L/L(sun) = [M/M(sun)]3.5

Den relativa livslängden blir (om vi antar att luminositeten är konstant under hela sjärnans livcykel, vilket inte är ett alltför dåligt antagande eftersom mesta tiden tillbringas på huvudserien) proportionell mot massan dividerat med luminositeten:

[M/M(sun)]/[M/M(sun)]3.5 = 1/[M/M(Sun)]2.5


M/M(sun)  L/L(sun)     Relativ livslängd    Livslängd
0.1       0.000316     316                  3160 GA
1         1            1                    10 GA
10        3160         0.00316              32 MA
100       10000000     0.0000100            0.1 MA
GA (giga annum) är miljarder år, MA är miljoner år
Den uppskattade livslängden stämmer väl sådär med de siffror du ger, men det är uppenbart att massiva stjärnor lever mycket kortare än solen och lättare stjärnor mycket längre. Det finns sofistikerade program som beräknar stjärnors utveckling, och jag antar att de approximativa siffror som ges i din lärobok kommer från sådana beräkningar.

Den springande punkten när det gäller livslängden är luminositeten - en tio gånger tyngre stjärna har inte tio gånger högre utan snarare 3000 gånger högre luminositet. Anledningen till detta är att den större gravitationskraften hos den större stjärnan ger högre temperatur och högre densitet i ett större område i centrum. Detta betyder att vätefusionen går snabbare och effektutvecklingen mycket större.



/Peter E

Nyckelord: stjärnors utveckling [15]; mass-luminositetsrelation [2];

1 http://csep10.phys.utk.edu/astr162/lect/binaries/masslum.html

*

Materiens innersta-Atomer-Kärnor [15452]

Fråga:
Vem var det som upptäckte fusionen (inte kall fusion utan vanlig fusion)? Vem kom på att det var fusion som var bränslet till reaktionen i stjärnor och vem började försöka exprimentera med fusion på Jorden?
/Simon J, Perstorp Gymnasium, Perstorp

Svar:
Ofta i vetenskapen kan man inte säga att det var en person som stod för en upptäckt. Man förstod att fusion av lätta kärnor var möjlig så snart man bestämt kärnmassorna för de lätta ämnena (masspektrograf, Aston c:a 1920). Man såg då att bindningsenergin var högre för vissa, och att man skulle kunna få ut energi med fusionsreaktioner.

Samma sak med nukleosyntes i stjärnor, men här är det klart att Eddington var den första som föreslog att fusion låg bakom. Detaljerna arbetades emellertid fram av andra, se länk 1. Se vidare fråga 14753 och länkar där.

Det första experimenterandet med fusion i stor skala på jorden var utvecklandet av vätebomben i början av 1950-talet. Här var Edward Teller en av förgrundsfigurerna, se länk 2. Experiment med kontrollerad fusion (tokamak mm, se fråga 12960) kom betydligt senare (1970-talet).
/Peter E

Se även fråga 14753 och fråga 12960

Nyckelord: fusion [17];

1 http://en.wikipedia.org/wiki/Stellar_nucleosynthesis
2 http://en.wikipedia.org/wiki/Teller-Ulam_design

*

Kraft-Rörelse [15605]

Fråga:
Det verkar vara så att naturen strävar efter att ha så låg potentiell energi som möjligt; bl.a. tyngdkraften drar allt till bottennivå. (Kan järn som har största massdefekten vara ett exempel på samma fenomen?) Finns det någon princip bakom detta? Någon regel som erfarenhetsmässigt gäller och/eller kan visas gälla med hänvisning till andra regler, som är mera grundlägande/postulatsliknande?
/Thomas Å, Arlandagymnasiet, Märsta

Svar:
Ja, järn/nickel har maximal bindningsenergi, och det är därför fusionen av element slutar där. Uppbyggnaden av de tyngre är en annan process som kostar energi. Se vidare grundämnen, bildandet av och bindningsenergi .

Principen är väl ganska enkel. Om vi har en potential så har vi även en kraft. En skidåkare som står uppe i backen påverkas av en kraft som vill få henne (Anja) ner i dalen. Anjas potentiella energi förvandlas först till rörelseenergi och sedan till värmeenergi genom friktion (bromsning). När hon väl är nere och i vila fordras det yttre energi (liftsystem eller bananer) för att hon skall kunna ta sig upp igen. Se potential/potentiell energi och framför allt fråga 13821 för mer än vad du någonsin vill veta om potential.

Nu är det inte alltid så enkelt som exemplet med skidåkaren. Ofta finns det någonting som hindrar att en kropp faller ner i ett potentialfält. Det finns t.ex. andra bevarande-lagar än totala energins bevarande. Vi har t.ex. rörelsemängdsmomentets bevarande i jordens rörelse runt solen. Även om solen gör sitt bästa att dra in jorden i sitt inre (och det är det efterstävansvärda tillståndet med lägst potentiell energi), så fortsätter jorden i sin bana eftersom den har en lagom stor rörelsekomponent vinkerätt mot riktningen mot solen.

I en atom kan en elektron befinna sig högt uppe i potentialgropen. Om det finns platser längre ner kan den "hoppa" till dessa lägre tillstånden. Förutsättningen är dels att det lägre tillståndet är ledigt (pauliprincipen ) och att "hoppet" är möjligt. Om detta är uppfyllt kommer den förr eller senare att ta sig ner till det lägsta lediga tillståndet.
/Peter E

Se även fråga 13821

Nyckelord: potential/potentiell energi [30];

*

Materiens innersta-Atomer-Kärnor [15929]

Fråga:
Hej! Vid betasönderfall bildas en elektronantineutrino, något som alltid redovisas då en neutron sönderfaller. Men när neutroner bildas, t ex vid fusionsprocesser, så redovisas inte att någon neutrin skulle delta för att fusionen skulla vara möjlig. Var kommer den ifrån? (Motsvarar neutrinen de flisor som blir när man klyver ved? De syns inte till vid uppbyggnaden av veden, men är en obligatorisk del vid vedhuggning - för att använda en bild i sammanhanget.)
/Thomas Å, Arlandagymnasiet, Märsta

Svar:
Hej Thomas! Neutriner bildas vid betasönderfall (svaga växelverkan) av en nukleon:

n -> p + e- + vanti

p -> n + e+ + v

Eftersom neutronen är tyngre än protonen går den senare reaktionen bara om energi finns tillgänglig, t.ex. i en atomkärna.

I kärnreaktioner bevaras normalt antalet neutroner och antalet protoner - en neutron kan frigöras, men den fanns redan i kärnan. Första steget i fusion av väte till helium är emellertid svag växelverkan, så där frigörs en neutrino:

p + p -> (2He) -> 2H + e+ + v
/Peter E

Nyckelord: neutrino [19]; kärnreaktion [5];

*

Blandat [16089]

Fråga:
Hur fungerar en luktlös doftspray?
/Veckans fråga

Ursprunglig fråga:
Hur fungerar en luktlös doftspray? En som man kan spraya med och odörer försvinner?
/Olivia R, Halmstad

Svar:
Olivia! Detta är knappast fysik, så vi vet mycket lite om det. Lite forskning på internet ger emellertid lite:

Ett av medlen som används är från ökenväxten Yucca_schidigera . Hur (och om) den fungerar vet man nog inte. I Wikipedia artikeln Air_freshener finns en sammanställning av hur doft-hämmare fungerar:

  1. Adsorption: aktiva ämnen (t.ex. aktivt kol, kiselgel) absorberar ämnena som luktar
  2. Kemisk neutralisering: en kemisk reaktion förstör det som luktar
  3. Desinfektion: lukter orsakas ofta av bakterieaktivitet som kan hämmas med desinfektion
  4. Maskering: många doftsprayer maskerar oönskade dofter med en starkare doft som är mer acceptabel
  5. Bedövning: luktsinnet bedövas
Metoderna 1, 2, 3 och 5 kan alltså i princip vara luktlösa. Metod 5 är tveksam eftersom den kan vara skadlig. Se även diffusion .
/Peter E

*

Energi [16133]

Fråga:
Hur fungerar solceller?
/Veckans fråga

Ursprunglig fråga:
Hej! Jag skriver ett arbete i skolan om solceller. När jag har letat information om hur solcellerna fungerar så står det först och främst att solcellen polariseras när den träffas av solljus så att framsidan blir negativt laddad och baksidan positivt laddad. Min fråga är då hur detta fungerar? Vad menar man med polariseras och hur polariseras solcellen enbart av solljuset?
/My S, österslätt, Karlshamn

Svar:
Hej My! För att förstå detta ordentligt måste man förstå halvledare och p-n övergångar (dioder). En bra förklaring på engelska finns på länk 1. Photovoltaic_cell , Photovoltaics och Solar_cell är lite mer avancerade.

Svenska Wikipedia (Solcell#Hur_en_solcell_fungerar ) säger:

En solcell är en typ av fotodiod. Solcellen består av två skikt: P-skikt och N-skikt. Det vanligaste ämnet i solcellen är kisel som har fyra valenselektroner. N-skiktet är sedan dopat med ett ämne med fem valenselektroner, exempelvis fosfor, och p-skiktet är dopat med ett ämne med tre valenselektroner, exempelvis bor. Alltså fattas det elektroner i p-skiktet, medan det blir extra elektroner i n-skiktet. Elektronkoncentrationerna är alltså olika på ömse sidor om kontaktskiktet. Diffusion leder då till att elektroner i n-skiktet vandrar över till p-skiktet. Det n-dopade skiktet blir positivt laddat, och det p-dopade skiktet bli negativt laddat, med ett starkt elektriskt fält däremellan. I mörker finns här inga fria elektroner.

Men kommer det en foton från solljuset ger den ifrån sig sin energi till elektronen och om fotonen har tillräcklig energi kommer elektronen att exciteras. När elektronen hamnar i det elektriska fältet mellan skikten, sveps den till det positivt laddade n-skiktet, där den kan ledas ut i en yttre krets (elledning).

Solceller tappar i verkningsgrad när temperaturen stiger. Det har visat sig att det är förhållandevis enkelt att använda passiv kylning och därmed minska förlusterna när temperaturen stiger.

De inkommande ljusfotonerna avlämnar alltså sin energi genom att lyfta en elektron från valensbandet (där elektronerna är bundna till en viss atom) till ledningsbandet (där elektronerna är fria att röra sig i hela kristallen.

Bilden nedan från Wikimedia Commons visar 14 MW Nellis Solar Power Plant som är Nordamerikas största fotovoltaiska anläggning och ligger i Nellis Air Force Base i Nevada. Panelerna styrs så att de alltid är vinkelräta mot solen.



/Peter E

Nyckelord: solenergi [14]; solcell [7];

1 http://science.howstuffworks.com/solar-cell.htm
2 http://news.mit.edu/2016/hot-new-solar-cell-0523

*

Materiens innersta-Atomer-Kärnor [16811]

Fråga:
hej:) jag håller på med mitt arbete i No och har fastnat lite, undrar om du kan hjälpa?

1. vilka olika ämnen skulle jag kunna få om en urankärna delade sig?

2. Hur många valenselektroner har Na?

3. varför är det svårt att få två protoner att sitta ihop? varför går det ändå?

4. vad krävs för att en elektron ska hoppa från ett inre skal till ett yttre?
/Jill l, lund

Svar:
Hej Jill! Bra, men ganska avancerade frågor!

1 Nästan alla medeltunga kärnor men utbytet är störst för masstal 90-110 och 130-150. Se nedanstående figur från Wikimedia Commons, Fission_product_yield .

2 Na har en valenselektron eftersom den finns i grupp 1 av det periodiska systemet, se WebElements .

3 Detta reds ut ganska detaljerat i fråga 12810 nedan. Två protoner kan inte bindas. Det som sker vid fusion i stjärnor är att diprotonen betasönderfaller (se bilden i fråga 10658 nedan) och bildar en deuteron.

4 Att det tillförs energi genom en kollision eller en foton.



/Peter E

Se även fråga 12810 och fråga 10658

Nyckelord: fission [15];

*

Materiens innersta-Atomer-Kärnor [16926]

Fråga:
Hej! Undrar främst om ni kan ge lite info om fusionens historia. var det så att det första experimenten egentligen var själva utvecklandet av vätebomben? och kom man på tanken om att man skulle börja utveckla fusionen som energikälla?
/Joakim F, Fredrika, Handen

Svar:
Man kände till fusionsreaktionerna tidigt på 1900-talet (när man kunde mäta atommassor insåg man potentialen att utvinna energi) och kunde studera dem tidigt på 30-talet när man hade acceleratorer. Här är en mycket bra sammanfattning från Wikipedia (Nuclear_fusion ):

Nuclear fusion occurs naturally in stars. Artificial fusion in human enterprises has also been achieved, although it has not yet been completely controlled. Building upon the nuclear transmutation experiments of Ernest Rutherford done a few years earlier, fusion of light nuclei (hydrogen isotopes) was first observed by Mark Oliphant in 1932; the steps of the main cycle of nuclear fusion in stars were subsequently worked out by Hans Bethe throughout the remainder of that decade. Research into fusion for military purposes began in the early 1940s as part of the Manhattan Project, but was not successful until 1952. Research into controlled fusion for civilian purposes began in the 1950s, and continues to this day.

När det gäller kontrollerad fusion som en energikälla brukar jag säga: När jag började studera kärnfysik i slutet på 1960-talet sa man att fusion är en kommersiell energikälla om trettio år. I dag säger man om femtio år! Tekniken är alltså betydlig svårare än vad man trodde från början! Vi får alltså till vidare nöja oss med solenergi som ju i grunden är fusionsenergi.

Se vidare fusion , Fusion och fråga 15452 nedan.
/Peter E

Se även fråga 15452

Nyckelord: fusion [17];

*

Materiens innersta-Atomer-Kärnor [17519]

Fråga:
Vi har jobbat med Atomfysik i skolan jag undrar om det går att fusionera olika ämnen med varandra, t.ex. en väteatom och en heliumatom? Och om det går, vad händer? Blir det fusionerade ämnet litium och ger väte och helium fusionen lika mycket energi som en fusion med väteatomer?
/Jim W, Storåskolan, Storå

Svar:
Jim! Nej, det går inte att fusionera 4He med en proton. Inte heller kan man slå ihop två 4He. Orsaken är att det inte finns några stabila nuklider med massan 5 eller 8.

Orsaken till detta är att 4He är väldigt stabil. Nedanstående figur visar bindningsenergin per nukleon för en massa kärnor. Du kan se att 4He är mycket mer bundet än de närmaste grannarna. Om man försöker göra t.ex. 5Li med en proton + 4He, så är det energetiskt fördelaktigt att gå tillbaka och återbilda 4He. Detta är också vad som sker.

Se vidare 14847 .



/Peter E

Nyckelord: bindningsenergi [23];

*

Energi, Materiens innersta-Atomer-Kärnor [17662]

Fråga:
Kall fusion på italienska
/Veckans fråga

Ursprunglig fråga:
Hej! Jag undrar vad du tror om den senaste rapporten om kall fusion? Vad är nytt jämfört med det som kom upp 1989?
/Arne S

Svar:
Det finns både likheter och skillnader. Likheten är att man presenterar nyheten på en presskonferens och förbigår den normala proceduren att publicera en fullständig rapport i en refereegranskad tidskrift. Den största skillnaden är att man föreslår en helt annan typ av fusion än vätefusion, nämligen fusion mellan väte och nickel. Mer om detta nedan.

Två italienska forskare, Focardi och Rossi, har den 14 januari 2011 demonstrerat en apparat som under en timme levererat en effekt på 12 kW med en input-effekt på 400 W. Hur apparaten är konstruerad säger man inte (av patentskäl), men den innehåller nickel och väte. Se NyTekniks artikel (länk 1) och nedanstående bild på apparaten (från länk 2).

Om funktionen hos apparaten verkligen är vad man hävdar är detta utan tvekan den största uppfinning som mänskligheten någonsin åstadkommit -- den skulle utgöra en i praktiken outsinlig källa till energi. Problemet är att det är ganska säkert inte sant eftersom det skulle kräva en fundamental ändring i fysikens lagar som vi uppfattar dem i dag. Detta är mycket likt Randell Mills idéer (fråga 14237 ) om ett nytt grundtillstånd i väte som kan utnyttjas som energikälla.

Uppgifter om data för demonstrationen är ganska vaga, men vi använder följande som approximationer för nedanstående överslagsberäkningar:

Tid för fortfarighetstillstånd i demonstrationen: 1 timme
Nettoeffekt (Put-Pin): 10 kW
Mängd väte förbrukat: 1 g
Flöde av vatten: 4.9 g/s

Vi antar dessutom i fortsättningen att sedan många decennier etablerade lagar i kärnfysiken gäller.

Artikeln av och Focardi och Rossi

Uppfinningen kallas energikatalysator (Energy Catalyser Boiler), men kunde även kallats evighetsmaskin. Det senare namnet kunde emellertid ställa till problem vid patentansökan .

Artikeln (länk 2) har refuserats i flera tidskrifter. I stället har författarna skapat en ny (webb)tidskrift kallad Journal of Nuclear Physics där ett antal märkliga artiklar publicerats.

Artikeln har flera av de i fråga 14237 listade kriterierna på pseudovetenskap. Mycket i artikeln är korrekta men triviala textboksfakta som t.ex. inledningen om bindningsenergi för atomkärnor och diskussionen om kvantmekanisk tunnling mot slutet.

Det som brister är emellertid beskrivning av apparaten. Det som i normala artiklar kallas experimentella procedurer saknas helt. Detta är helt i strid med det vetenskapliga arbetssättet (fråga 14237 ): alla experimentella resultat skall kunna reproduceras. Vem som helst skall alltså kunna upprepa experimentet och få samma resultat, men detta kan man ju inte göra utan en detaljerad beskrivning.

I diskussionen mot slutet i artikeln förklaras barriärpenetreringen med någon effekt att elektron och proton kommer mycket nära varandra. Elektronen skulle då skärma protonens laddning och protonen skulle därmed inte utsättas för en repulsiv coulombbarriär. Det skulle kanske kunna bli en ny film: Honey, I Shrunk the Hydrogen Atom (jämför Honey,_I_Shrunk_the_Kids ), men någon ny energikälla är det knappast.

Det finns andra märkligheter i artikeln där författarna visar att deras kunskaper i kärnfysik är mycket begränsade. I stället för att i detalj dissikera artikeln och utvärderingsrapporten (som också finns under länk 2) gör vi några enkla beräkningar.

Kemisk energi från väte

Energipotentialen från väte (den energi man maximalt kan få ut av en massenhet väte genom kemiska reaktioner) är enligt figuren i fråga 17516 drygt 140 MJ/kg. Från ett gram väte kan man alltså få ut 140 kJ. Om vi dividerar detta med den utvecklade medeleffekten får vi

(140 kJ)/(10 kJ/s) = 14 sekunder.

1 g väte skulle alltså räcka i 14 sekunder - långt från en timme som demonstrationen varade. Kemiska reaktioner med väte kan alltså inte förklara energiutvecklingen.

Strålning från fusion

Den mest förekommande nickelisotopen är 58Ni. Den dominerande fusionsreaktionen bör då vara:

1H + 58Ni -> 59Cu

Denna reaktion har Q-värdet 3.4 MeV. Det betyder att den exciterade 59Cu kärnan måste på något sätt göra sig av med 3.4 MeV för att hamna i sitt grundtillstånd. Det enda kända processen för detta är genom gammasönderfall. 59Cu är radioaktiv och sönderfaller med halveringstiden 80 s med b+-sönderfall:

59Cu -> 59Ni + e+ + v

Q-värdet för sönderfallet är 4.8 MeV. Totala utvecklade energin per reaktion är då 3.4+4.8=8.2 MeV. Detta värde är helt i linje med den maximala bindningsenergin per nukleon i figuren i fråga 1433 på drygt 8 MeV.

59Ni är mycket långlivat så vi kan bortse från det sönderfallet. Vi bortser även att en del energi försvinner ut ur systemet i form av neutriner.

Antal fusionsreaktioner som krävs per sekund för att generera 10 kW:

(10*103 [J/s])/(8.2 MeV * 1.6*10-13 [J/MeV]) = 0.8*1016 /s

För varje fusion med 58Ni får vi även ett sönderfall av 59Cu. Vi har alltså minst 2*0.8*1016=1.6*1016 gammasönderfall/s.

Aktiviteten blir 1.6*1016/(3.7*1010)=4.3*105 Ci.

Detta är en enorm aktivitet. I Curie sägs att

A radiotherapy machine may have roughly 1000 Ci of a radioisotope such as cesium-137 or cobalt-60. This quantity of nuclear material can produce serious health effects with only a few minutes of exposure.

Den visade apparaten är alldeles för liten för att innehålla tillräckligt med strålskydd. Alla närvarande borde alltså ha fått en dödlig stråldos. Man har dessutom mätt med strålningsdetektorer och konstaterat att strålnivån inte överstiger bakgrundsnivån.

Ett annat problem är att om en stor andel av gammastrålningen slipper ut, så blir det för liten effekt för att skapa apparatens påstådda positiva nettoeffekt.

Strålskydd/infångande av energi

Om fusionen sker med etablerade lagar så kommer energin i huvudsak ut i form av gamma-strålning. Låt oss se hur mycket strålskydd man behöver för att fånga in så mycket strålning att det inte är farligt att gå nära apparaten. Observera att man måste ändå stoppa det mesta av strålningen för att kunna ta tillvara energin.

Halveringstjockleken för bly för gammastrålning med energin 2-4 MeV är c:a 20 g/cm2. Med blys densitet 11 g/cm3 blir halveringstjockleken i cm

20 g/cm2/(11 g/cm3) = 2 cm

Med aktiviteten ovan på 4*105 Ci och en rimlig säker nivå på 1 mCi (som en vanligt lab-preparat) får vi en absorptionsfaktor på

1*10-6/(4*105) = 2.5*10-12

Vi antar vi behöver x halveringstjocklekar:

2-x = 2.5*10-12

(-x)*log10(2) = log10(2.5) -12

(-x)*0.30 = 0.40 -12 = -11.6

x = 11.6/0.3 = 39

Vi behöver alltså ett blyskydd på 39*2 = c:a 80 cm för att få ett säkert strålskydd! Detta hade man uppenbarligen inte vid demonstrationen!

Energi från fusion

Antag att vi har 0.8*1016 fusionsreaktioner per sekund (se ovan). 1 g väte är 1 mol väte. 1 g väte innehåller 6.022*1023 väteatomer. Konstanten är Avogrados tal. 1 g väte räcker då i

6*1023/(0.8*1016) = 7.5*107 s = 21000 timmar. Energipotentialen i 1 g väte vid fusion är alltså mer än tillräcklig.

Demonstrationen av nettoeffekten på 10 kW

Apparaten värmer vatten från 13oC till 100oC (DT=87 K). Enligt fråga 14203 är vattens specifika vämekapacitet 4.18 J/gK. Sedan förångas vattnet (kräver 2260 J/g). Med vattenflödet 4.9 g/s får vi den utvecklade effekten

(4.9 [g/s])*(4.18 [J/(g*K)]*87 K + 2260 [J/g]) = (1.8+11.1)*103 = 12.9 kW

vilket är nära den påstådda effektutvecklingen.

Sammanfattning och slutsats

Om den uppvisade apparaten skulle fungera krävs en fullständig revision av våra kunskaper om atomkärnor. Artikeln är dessutom av pinsamt låg kvalité med många tecken på pseudovetenskap. Att artikeln refuserats i flera tidskrifter visar bara att refereesystemet fungerar bra.

Det finns tre alternativ för den s.k. energikatalysatorn:

  1. Den fungerar som beskrivet och uppfinnarna blir rika som troll (osannolikt)
  2. Man har gjort ett oavsiktligt misstag (knappast, eftersom den påstådda effekten är mycket stor)
  3. Det är ett medvetet bedrägeri för att lura till sig riskkapital (troligaste; detta är numera tyvärr ganska vanligt)

Låt oss fundera på om den utvecklade effekten på 10 kW är rimligt. En normal spisplatta utvecklar c:a 1 kW. När den varit igång en stund är det lätt att känna värmen från plattan om man står inom c:a 1 meter. Effekten 10 kW (som ju måste ta vägen någonstans) borde vara märkbar för alla som var i rummet vid demonstrationen - det borde blivit varmt som i en bastu! Inget sägs emellertid om att det blev varmt i rummet.

Vart tog energin vägen då? Lagrades i apparaten? Nej, det finns inget material med så hög specifik värmekapacitet att det skulle vara möjligt. Försöket är alltså inte enbart tvivelaktigt vad gäller våra kunskaper i kärnfysik. Det verkar även som om apparaten strider mot fysikernas heligaste lag: lagen om energins bevarande.

Man skall inte avslöja trollkarlens trick, men jag har ett förslag till lösning av mysteriet. Det sägs ingenstans, och syns ingenstans i videor och på bilder vad man gör med ångan som genereras. Om denna kondenseras inne i apparaten och släpps ut som vatten återfås ångbildningsvärmet. Enligt ovan används huvuddelen av effekten till att förånga vattnet, så återvinning skulle göra att man kan få en effektbalans utan att blanda in fusion.

Det finns många saker som tyder på att energikatalysatorn inte kan fungera som man hävdar. För mig är de viktigaste problemen avsaknaden av tydlig netto-effektutveckling och avsaknaden av gammastrålning.

Energiutveckling:
Vart tog den utvecklade energin (i första hand i form av vattenånga) vägen under demonstrationen? Kan man verkligen vara säker på att allt kylvatten förångades? Den mesta energin upptas genom ångbildningsvärmet, så det är helt avgörande att visa att allt vatten i form av vattenånga tas ut ur systemet.

Kärnfysikproblem:
I artikeln (länk 1) sägs att det är frågan om fusion mellan nickel och väte till isotoper av koppar. Man hävdar även (utan någon beskrivning av metoden) att man detekterat ett från det naturliga förhållandet avvikande värde på isotopförhållandet för kopparisotoper.

Var och en med elementära kunskaper i kärnfysik kan se ett antal problem med Rossis demonstration och beskrivning.

  1. I tabell 3 i Rossis artikel anges den totala utvecklade energin (Q-värdet) för vätefusion med 58Ni till 41.79 MeV. Detta är totalt felaktigt, det korrekta värdet är 8.2 MeV. Det senare värdet är i god överenskommelse med bindningsenergin per nukleon i detta område, se figur 1 i artikeln.

  2. Coulomb-barriären för en proton mot nickel är av storleksordningen 1 MeV. Transmissionen beräknad med etablerade kvantmekaniska metoder är nästan noll - ett faktum som även Rossi konstaterar i artikeln. Det är svårt att se hur den kemiska miljön skulle väsentligt kunna påverka kärnans Coulomb-barriär.

  3. Även om man accepterar att barriärpenetrationen inte är något problem så är det svårt att se hur den resulterande kärnan överför sin överskottsenergi till omgivningen utan gammastrålning.

  4. Det finns absolut inget skäl att den bildade radioaktiva 59Cu-kärnan skulle sönderfalla på ett helt annorlunda sätt än vad som är väl etablerat. Man borde alltså detektera gammastrålning och annihilationsstrålning.

  5. I fusionen med 58Ni bildas en stor aktivitet av 59Cu (av storleksordningen 1016 Bq, se ovan) som b+-sönderfaller till 59Ni med en halveringstid på 82 sekunder. I artikeln sägs att "No radioactivity has been found also in the Nickel residual from the process". Med tanke på den mycket höga aktivitet som måste ha producerats när energikatalysatorn kördes är detta uttalande ytterst förvånande: det borde vara lätt att verifiera produktionen av 59Cu genom att detektera gammastrålning.

Diskussion
För nästan exakt 100 år sedan (7 mars 1911) presenterade Rutherford sin modell av atomen med en mycket liten kärna (10-15 m) som innehåller nästan hela atomens massa och elektroner som rör sig omkring kärnan inom ett område på c:a 10-10 m. Sedan dess har ett stort antal experimentalister med sofistikerad utrustning och teoretiker med kraftfulla datorer studerat atomkärnans egenskaper. I dag måste man säga att vi förstår atomkärnan mycket väl. Det är svårt att tro att 100 års forskning om atomkärnan är så bristfällig som punkterna ovan indikerar.

Vad gäller tunnlingen skulle man kunna tänka sig att elektroner på något sätt skärmar barriären och släpper in protonen. Problemet med detta är att Heisenbergs obestämdhetsrelation förbjuder elektroner att vistas en längre tid i ett så litet område som atomkärnan. Denna förklaring skulle alltså på ett grundläggande sätt förändra kvantmekaniken som vi känner den.

En annan förklaring av tunnlingen som framförts (Widom-Larsen Theory Portal ) är att protonen växelverkar med en elektron och förvandlas till en neutron och en neutrino (neutronen är ju neutral och har inga problem att ta sig in i kärnan):

p + e- -> n + v

Denna reaktion är fullt tillåten, men problemet är att den sker med den svaga växelverkan och har därmed en mycket liten sannolikhet. Om man vill använda ovanstående reaktion i förklaringen måste man väsentligt modifiera den väl etablerade teorin för den elektrosvaga växelverkan. Dessutom borde man vänta sig att en del av de bildade neutronerna "smiter ut". De skulle då reagera med omgivningen och ge upphov till lätt detekterbar gammastrålning.

I punkterna 3, 4 och 5 ovan är problemet att man i Rossis försök inte observerar någon gammastrålning. En förklaring som framförts är att gammastrålningen från kärnan på något (magiskt?) sätt förvandlas till värmestrålning. Detta är något som aldrig observerats och det står helt i strid med vad vi vet om elektromagnetisk strålning.

Bakgrundsmätning
Om man verkligen vill visa att energikatalysatorn fungerar borde man göra en blind bakgrundstest: Enligt Rossis artikel är vätet nödvändigt för att apparaten skall producera energi. Låt en oberoende person kontrollera väteflödet utan att konstuktörerna vet om flödet är på eller av. Kontrollera att energiproduktionen är fullständigt korrelerad med vätetillförsel.

Låt oss avsluta med ett citat från Carl Sagan: Extraordinary claims require extraordinary evidence.

Bra sammanfattning av Göran Ericsson, Uppsala universitet: Kall fusion i Italien
En uppdaterad diskussion av Peter Ekström: Kall fusion på italienska
Kjell Alekletts synpunkter och diskussionsforum (på engelska): Rossi energy catalyst – a big hoax or new physics? .



/Peter E

Nyckelord: kall fusion [8]; pseudovetenskap [11]; nyheter [11];

1 http://www.nyteknik.se/nyheter/energi_miljo/energi/article3073394.ece
2 http://fragelada.fysik.org/resurser/Rossi-Focardi_paper.pdf

*

Kraft-Rörelse [17710]

Fråga:
Varför har tävlingsbilar kväve och inte luft i däcken?
/Veckans fråga

Ursprunglig fråga:
Tävlingsbilar har kväve i däcken för att minska tryckökningen när däcken blir varma. Kväve borde väl expandera ung lika mycket som vanlig luft när däcken blir varma? Luft innehåller fukt, men har det verkligen betydelse i detta fall? Hur stor betydelse i så fall? Och varför?

Mvh Niclas
/Niclas P, Rudbeckskolan, Sollentuna

Svar:
Niclas! Det finns mycket diskussion om nytta eller inte nytta med kväve i däck, se t.ex. länk 1 nedan, Eftersom luft är nära 80% kväve är det klart att effekten av att fylla däcken med kväve i stället för luft är mycket marginell. Det låter alltså för den vanlige bilföraren som en bluff för att lura pengar av oss - tänk att till höga priser kunna sälja i stort sett luft!

För vissa speciella tillämpningar kan det nog ha en liten betydelse, t.ex. för ett flygplansdäck som blir överhettat. Man kan även tänka sig att gummit med tiden oxideras av syret och därmed blir sprött. Jag tror emellertid att mönstret slits ut innan man märker av effekten av oxidation.

Att syret skulle läcka ut snabbare måste vara en liten effekt. Syremolekylen är visserligen lite mindre än kvävemolekylen (eftersom samma skal fylls och kärnladdningen är högre för syre). Detta motverkas av att diffusionshastigheten är högre för en lättare molekyl (kväve). Skulle man se någon effekt i lufttrycket så måste hälften av syret läcka ut, och det tror jag inte på. Dessutom kontrollerar väl varje bilförare däcktrycket regelbundet !

Att kvävet skulle ha något med tryckökningen när temperaturen blir högre är också nonsens. Från allmänna gaslagen kan man härleda (se fråga 15619 )

Dp/p = DT/T

Alltså den relativa tryckändringen beror endast av temperaturer, inte av vilken gas man har! Det är emellertid sant som du säger att man använder kvävgas till däcken i tävlingsbilar. Det kan knappast vara läckage (däcken på en Formel 1 bil varar högst någon timme). Knappast oxidation heller. Möjligen lite säkrare vid brand, men Formel 1 bilar brinner sällan i dag. Förklaringen ges i länk 2.

Kvävet innehåller alltså ingen vattenånga eftersom det framställs genom att kyla ner luft tills kvävet kondenseras till vätska, och vattnet fryser bort på ett tidigt stadium. Vatten ställer till det i däcket eftersom det kan kondensera till vätska vid låg temperatur, men blir ånga vid hög temperatur. Vid hög temperatur har vi alltså fler molekyler (luft+vattenånga) och därmed högre tryck. Gaslagen säger ju

pV = nRT,

så om antalet mol n ökar så ökar trycket. Man kan se det så att eftersom n ökar med temperaturen (fler molekyler i gasfas) blir sambandet p-T olinjärt.

Användandet av kväve är alltså ett villospår, och det är som du föreslår vatteninnehållet som är det viktiga. Man kan lika gärna använda luft så länge den inte innehåller något vatten. Tydligen är det emellertid enklare att framställa kvävgas än torr luft!
/Peter E

Nyckelord: gaslagen, allmänna [24];

1 http://www.straightdope.com/columns/read/2694/is-it-better-to-fill-your-tires-with-nitrogen-instead-of-air
2 http://auto.howstuffworks.com/question594.htm

*

Materiens innersta-Atomer-Kärnor [17758]

Fråga:
Hej! Vi håller på att jobba med kärnfysik och diskuterat bindningsenergier kopplat till fission/fusion. Jag vet ju att det är massdefekten som hjälper oss att få ut den frigjorda energin vid dessa processer. Men hur kan man säga att vi går från ett högre till ett lägre energitillstånd när vi går från kärna med låg bindningsenergi till en med högre bindningsenergi? Som exempel; vi klyver en tung kärna som U-235 i två ungefär lika stora delar. Delarna har högre bindninsenergi/nukleon dvs. det år åt mer energi att hålla ihop fissionsprodukterna än U-235 (eller?). Och ändå kan vi få ut energi. Borde inte energin från massdefekten "gå åt" till ökad bindningsenergi i fissionsprodukterna? Snälla, var är det jag tänker galet? En gång i tiden har jag ju förstått det här...
/Fredrik O, Hulebäcksgymnasiet, Mölnlycke

Svar:
Hej igen Fredrik! Det beror på definitionen av bindningsenergi, se fråga 1433 .

I nedanstående figur visas bindningsenergin per nukleon som funktion av masstalet A=Z+N. Utgångspunkten för definitionen av bindningsenergin är Z väteatomer och N neutroner. Dessa har då definitionsvis bindningsenergin = 0, dvs en linje sammanfallande med den horisontella axeln. När man sätter samman dessa beståndsdelar till en kärna AZ och då frigörs energi. Vi hamnar då på den grå linjen i diagrammet. Ju större bindningsenergi som frigöres, desto mer bunden är kärnan.

Det hade varit bättre om man definierat bindningsenergin som negativ. Då hade de mest bundna kärnorna hamnat längst ner, vilket intuitivt hade varit met tillfredsställande. Se det så här: bindningsenergin är den energi som krävs för att plocka isär kärnan helt och hållet. Om vi definierar energin av den sönderplockade kärnan som 0, blir alltså energin hos den sammansatta kärnan negativ. Av praktiska skäl definierar man dock bindningsenergin som den energi som krävs för att plocka isär kärnan, dvs som positiv.



/Peter E

Nyckelord: bindningsenergi [23];

*

Materiens innersta-Atomer-Kärnor [17799]

Fråga:
Jag har lite funderingar om fusion som jag vill ha klarhet i. Det är mest tankar som är tillsnurrade nu när vi arbetar med astronomi och elevfrågorna blir lite mer komplicerade. Jag har försökt läsa mig till faktan men har inte lyckats få någon helhet än.

I solen sker fusion där protoner (vätekärnor) slås samman till heliumkärnor. Något som jag tidigare inte reflekterat över är att det då blir fel att säga att det är väteatomer som slås samman till heliumatomer eftersom elektronerna inte finns inblandade.

Första frågan är då var elektronerna finns. Min egen tanke är att solens inre är såpass varmt att elektronerna exiterats bort från protonerna helt och hållet eftersom de väl vid solens födelse, då partikelmolnet drog ihop sig, fanns runt atomkärnan? Var i solen finns de isf under pågående fusionsprocess?

Nästa tanke är då hur ljuset i solen blir till eftersom ljus utsänds då exiterade elektroner hoppar tillbaka till sina ursprungliga "skal" runt atomkärnan.

Hur och när binds elektronerna till sina kärnor i en supernova då tyngre ämnen än järn bildas?

Tack för en superbra frågelåda!
/Petri M, Mariefreds skola, Marefred

Svar:
I solens centrum där fusionsreaktioner sker är det c:a 15 miljoner K. Vid så hög temperatur kan inte elektronerna bindas till kärnorna utan man har ett plasma, vilket är en gas av laddade partiklar, positiva joner och negativa elektroner, se Plasma_(physics) .

Elektronerna måste befinna sig på samma ställe som jonerna. Om joner och elektroner haft olika fördelning skulle det bildas elektriska fält som snart jämnat ut fördelningen igen.

Ljus uppkommer även från laddade partiklar som kolliderar (accelereras), inte bara från atomära övergångar i atomer.

Eftersom den elektriska kraften har lång räckvidd och är mycket stark är det inget problem för kärnorna som bildats att fylla sina elektronskal. Se fråga 967 för en beskrivning av vad som händer med elektronerna vid normalt radioaktivt sönderfall.

Detaljer om stjärnors energiproduktion och utveckling finns i fråga 10658 och länkar därifrån.

Tack!
/Peter E

Nyckelord: solens energiproduktion [9];

*

Blandat [17930]

Fråga:
Hej! Under rubriken fysikalisk förståelse nämns att olika fysikaliska lagar har olika status, Newtons lagar står över Keplers, ty de senare kan härledas ur de senare. Därmed kan antalet fundamentala lagar hållas på rimlig nivå. Vilken är denna rimliga nivå? Dvs vilka är dessa lagar, finns de listade? ("En naturens lagbok/grundlagsbok.")
/Thomas Å, Knivsta

Svar:
Thomas! Intressant idé, men någon sådan lagbok finns tyvärr inte. Den hade kanske behövts för att avfärda de mest befängda påståendena, t.ex. kall fusion (17662 ).
/Peter E

*

Universum-Solen-Planeterna [18095]

Fråga:
Hej!

Såhär långt är jag med: I en stjärna sker fusion som smälter samman protoner till tyngre ämnen. I vår sol sker fusionen mellan väte och helium. Allt eftersom de lättare ämnena försvinner ökar trycket i stjärnans centrum vilket ger en ökad temperatur som i sin tur startar nya fusionsprocesser som tillåter tyngre ämnen att bildas. Jag har sett flera program (t.ex. Wonders of the solar system, Wonders of the universe) där man säger att då järn bildats i en stjärna kommer gravitationen inte längre orka stå emot det enorma trycket ut vilket skapar en supernova. Det ska tydligen ske rätt snabbt efter att järnfusionen startat.

Detta vill jag gärna ha klarhet i: 1. I vår sol finns redan järn, i liten mängd. Är det ifrån solsystemets början då stoffmolnet drog ihop sig? 2. Är järnet i form av plasma? 3. Finns järnet i kärnan av solen? 4. Vad är det som gör att den, förhållandevis stora kärnan inte råkar ut för en partikel och bildar ett tyngre ämne?

Mvh Petri
/Petri M, Mariefreds skola, Mariefred

Svar:
Petri!

1 Järnet fanns från början. Det är först på senare stadier som fusion till järn förekommer och då i något tyngre stjärnor än solen.

2 Ja, även på solens yta är temperaturen så hög att atomerna är delvis joniserade.

3 Nej, det är jämnt utspritt.

4 Temperaturen är inte tillräckligt hög för att åstadkomma fusion i tyngre kärnor.
/Peter E

*

Materiens innersta-Atomer-Kärnor [18254]

Fråga:
Hej, jag undrar om vi kommer ha samma problem med radioaktivt avfall om vi utvecklar kall fusion?
/Anna K

Svar:
I klassisk kall fusion, se fråga 2409 är eventuell produktion av tritium det största problemet. För andra kall fusion varianter (s.k. LENR, Low Energy Nuclear Reactions) kan man tänkas producera radioaktiva isotoper, men oftast ganska kortlivade som kan hanteras ganska lätt.

Problemet är att det är osannolikt att kall fusion verkligen fungerar. Det finns flera mycket goda skäl till att kall fusion inte finns, se fråga och 2409 och 17662 .
/Peter E

Nyckelord: kall fusion [8];

1 http://science.howstuffworks.com/starships-use-cold-fusion-propulsion.htm

*

Materiens innersta-Atomer-Kärnor [18298]

Fråga:
Hej! Det har producerats isotopen Helium-2, dvs två protoner so håller ihop!, med en halveringstid(?) på ~10^-27 s och ett sönderfall till väte-2.

Hur sker det sönderfallet? En plusladdning försvinner och en neutron skapas. Energibalansen, hur är den? Skapas neutriner eller t o m absorberas från omgivningen? Och kan man verkligen säga att en isotop existerar när det gäller så korta tidrymder? (Beklagar den flerdelade frågan, men den rör ju ett och samma ämne.)
/Thomas Å, Knivsta

Svar:
Halveringstiden för diprotonen är nog lite längre än vad du säger, men systemet är obundet och sönderfaller till två protoner i princip så snabbt det går.

Det alternativa sönderfallet är ett vanligt betasönderfall, se fråga 15929 . Betasönderfallet har en enormt mycket lägre sannolikhet och förekommer i princip endast centrum av stjärnor. I stjärnor kolliderar protonerna hela tiden, och i en mycket liten andel av dessa kollisioner sker även ett betasönderfall till deuterium. Det är första steget i denna s.k. proton-proton kedja som reglerar hastigheten hos vätefusionen i stjärnor, se Proton-proton_chain_reaction .

Diprotonen har även observerats i ett par exotiska sönderfall, se Diproton .

Anledningen till att diprotonen är obunden är inte, som man skulle kunna tro, att protonerna repellerar varandra. Det gör de naturligtvis, men det är en i sammanhanget liten effekt. Den avgörande effekten är spinnberoendet hos kärnkraften som håller ihop neutroner och protoner i atomkärnorna, se Isotopes_of_helium#Helium-2_(diproton) .

Deuteronen (se nedanstående figur) har i sitt grundtillstånd spinn/paritet 1+. Detta åstadkoms genom att neutronens och protonens spinn (1/2) är riktade åt samma håll. Det lägsta exciterade tillståndet i deuterium, med spinnen antiparallellt, ligger över separationsenergin på 2.2 MeV och är alltså obundet.

För fallen diproton och dineutron är tillstånden med parallellt spinn förbjudna enligt pauliprincipen (Pauli_exclusion_principle ), eftersom två identiska fermioner (halvtaligt spinn) inte kan ockupera ett tillstånd med identiska kvanttal. Bosoner (partiklar med heltaligt spinn) behöver däremot inte lyda pauliprincipen.

Det som alltså orsakar att diprotonen, dineutronen och det första exciterade tillståndet i 2H är obundna är alltså att kraftverkan mellan två nukleoner med parallellt spinn är större än när spinnen är antiparallella. Om denna senare kraftverkan bara hade varit lite starkare så hade diprotonen varit bunden. Man hade då fått en helt ohämmad fusion av protoner så att stjärnor knappast hade kunnat bildas.

Se även fråga 20615 .



/Peter E

Nyckelord: bindningsenergi [23]; kärnkrafter [7]; pauliprincipen [10];

*

[18300]

Fråga:
Hej! Vid fusioner, som i solen, skapas neutriner i massor och vid t ex betasönderfall bildas antineutriner. Finns det några reaktioner som kräver neutriners/antineutriners närvaro, och som så att säga skulle vara "neutrinoätande reaktioner"? (Finns neutrinokonsumenter?)Om inte kommer väl antalet neutriner/antid:o att bara öka och utgöra en allt större del av universums massa och energi(pga hastigheten)?
/Thomas Å, Knivsta

Svar:
Thomas! Vi har diskuterat detta i fråga 17942 .

*

Materiens innersta-Atomer-Kärnor [18316]

Fråga:
Hejsan! Jag har en fundering om fusion, eller vad man nu kan kalla detta. Är det på något sätt möjligt, att bryta elektronskalen i en atom, eller skapa instabilitet så att elektronerna dras till protonerna och bildar massa neutroner? För minus och plus dras ju till varandra. Visserligen skulle det nog ta väldigt mycket energi, men det blir väl energi från denna fusion? Min lärare kunde nämligen inte svara på denna fråga, så därför ställer jag den till er istället :) Tack på förhand!
/Simon H, Fagrabäckskolan, Växjö

Svar:
Hej Simon! Ja, man får nog kalla det fusion, kall fusion eller LENR (Low Energy Nuclear Reactions).

Det du beskriver att en elektron fångas in av en proton och bildar en neutron är mycket likt en teori framförd av Widom och Larsen, se länk 1 och figuren nedan.

En fri neutron sönderfaller (länk 2) enligt

n --> p + e- + vanti, Q=782 keV

Man kan naturligtvis vända på reaktionen och få elektroninfångning

p + e- --> n + v, Q=-782 keV

Det finns emellertid ett antal problem med teorin, och de flesta kärnfysiker tror inte det ligger något i den. Det viktigaste argumentet mot teorin är att Q-värdet för elektroninfångning är negativt. Det betyder att elektronen behöver ha en energi på över 782 keV för att reaktionen skall vara energetiskt möjlig. Det är mycket svårt att se hur detta kan ske under normala förhållanden. (Det kan ske i neutronstjärnor, se fråga 17998 , men det är en annan sak.)

För att elektronen och protonen skall kunna reagera måste de vara mycket nära varandra. Sannolikheten för detta i en så liten kärna som 1H är mycket liten. Eftersom reaktionen dessutom sker med den svaga växelverkan blir sannolikheten för en reaktion mycket liten.

En experimentell invändning mot modellen är dessutom att om det bildades neutroner så borde man kunna detektera dessa. Det har man inte gjort.

Widom-Larsen antar att det bildas någon slags "tung elektron" som skulle kunna reagera enligt ovan, men det finns inga som helst bevis på att sådana tunga elektroner existerar.



/Peter E

Nyckelord: kall fusion [8];

1 http://newenergytimes.com/v2/sr/WL/WLTheory.shtml
2 http://nucleardata.nuclear.lu.se/nucleardata/toi/nuclide.asp?iZA=1

*

Universum-Solen-Planeterna [18495]

Fråga:
Hej! Vi undrar vad som är den största skillnaden mellan en stjärna och en planet? Och sen har vi en fråga till, vad tycker du är den största obesvarade gåtan inom stjärnor? Skulle vara tacksamma om du kunde svara ganska fort, vi är väldigt nyfikna om det här med stjärnor!
/Charlotta och Moa J, Linköping

Svar:
Hej Charlotta och Moa! En stjärna är en kropp som huvudsakligen består av väte och helium och som är tillräckligt stor för att temperaturen i centrum gör kärnreaktioner möjliga.

Wikipedia säger:

En stjärna är en mycket stor och självlysande himlakropp av plasma. Den närmaste stjärnan sett från jorden är solen.

En planet är en himlakropp som rör sig i en omloppsbana runt en stjärna, har tillräckligt stor massa för att vara nästintill rund, och dominerar sin omgivning. Om massan är så stor att kärnfusion sker, så räknas himlakroppen dock som stjärna och inte planet. Se fråga 14788 .

Även om vi bara har kunnat studera stjärnor på avstånd så förstår vi tack vare våra kunskaper i fysik ganska väl hur de fungerar. Jag kan inte komma på något specifikt problem med stjärnor, men det mest aktuella problemet man sysslar med är att upptäcka planeter kring andra stjärnor (s.k. exoplaneter) för att kunna förstå exakt hur planetsystem bildas.

Se vidare stjärnors utveckling .
/Peter E

Nyckelord: planet [17]; exoplaneter [17]; stjärna [4];

*

Materiens innersta-Atomer-Kärnor [18606]

Fråga:
Hej jag har ganska många frågor.

1.En kärna av uran-235 träffas av en neutron. En fission inträffar. efter den har urankärna och neutronen bildat två nya atomkärnor och två fria neutroner. Den ena nya kärna är cesium-137. vilken är den andra?

2. De två atomkärnor 2H och 3H, slås samman.
a. vilket ämne är det före sammanslagningen?
b. efter kärnreaktionen finns en fri neutron och en atomkärna. Hur kan denna skrivas?
c. Vilket ämne är det alltså?
/Sara a, Runby, UpplandsVäsby

Svar:
1 Antalet protoner/neutroner före och efter fissionen bevaras:

protoner:
före 92 (atomnumret för U)
efter 55 (atomnumret för Cs)
differens 92-55=37 vilket är Rb

neutroner:
före (1+235-92)=144
efter (137-55)+2=84
differens 60

masstalet för den andra slutkärnan (37+60)=97

den andra slutkärnan är alltså Rb-97

2 a Båda kärnorna är väte (H)
b 2H + 3H -> 4He + n
c Helium bildas. Detta är den mest lovande fusionsreaktionen.
/Peter E

Nyckelord: fission [15]; fusion [17];

*

Materiens innersta-Atomer-Kärnor [18618]

Fråga:
Hej! Jag arbetar med ett projekt i Fysik B som handlar om magnetiska fält och fusion. Så som jag har förstått det så kan man stänga in plasmat med hjälp av magnetiska fält (s.k. magnetisk flaska), men det som jag inte riktigt förstår är hur atomkärnorna ska kollidera. Kärnorna är ju båda positivt laddade så de kommer att färdas åt samma håll, hur kan de då kollidera med varandra? Borde de inte färdas mot varandra för att kunna göra det?
/Sandra Y

Svar:
Sandra! Jovisst måste kärnorna färdas mot varandra med hög hastighet för att man skall få en kärnreaktion. Det är därför man behöver en hög temperatur hos plasmat. Hög temperatur betyder hög hastighet i slumpmässiga riktningar. En liten del av kärnorna kommer då att vara på kollisionskurs. Se vidare fråga 14847 .

Det är alltså den slumpmässiga rörelsen (temperaturen) som ger upphov till fusion. Du har dock rätt i att det även finns en ström runt torusen. Denna ström induceras genom att ett varierande magnetfält genom hålet i torusen inducerar en ström. Denna ström ger genom kollisioner (resistans) en uppvärmning av plasmat, se Tokamak#Ohmic_heating .

Det är alltså ganska komplicerade magnetfält (konstanta och varierande) som används i en Tokamak, se nedanstående bild från Tokamak .



/Peter E

Nyckelord: fusion [17];

*

Energi [18782]

Fråga:
Kan man skapa något ämne som ger mer energi än kärnkraft?
/Veckans fråga

Ursprunglig fråga:
Kan man skapa något ämne som ger mer energi eller är kraftigare än kärnkraft?
/Anette J, Gamla Uppsala skola, Uppsala

Svar:
Bra fråga som faktiskt inte tycks vara besvarad här!

Energiproduktion är en process där man vinner energi genom att massan i sluttillståndet är mindre än massan i begynnelsetillståndet. Differensen i massa ger genom Einsteins formel E=mc2 en energi som kan uttnyttjas.

För mekaniska och kemiska energikällor är det inte meningsfullt eller brukligt att tala om en mass-skillnad eftersom den är omätbart liten, se fråga 17491 . För kärnreaktioner är mass-skillnaden emellertid fullt mätbar.

För traditionell kärnenergi (fission av uran) är mass-skillnaden ungefär 0.3%. För fusion (sammanslagning av lätta ämnen) är mass-skillnaden maximalt 0.7%.

För att få en energikälla gäller det alltså att hitta en process där sluttillståndet har mycket mindre massa (= energi) än begynnelsetillståndet.

Det finns bara en känd process som ger bättre energiutbyte än fusion: att låta en massa falla ner i ett svart hål. Då kan man teoretiskt utvinna 50% av vilomassan som energi, se fråga 14367 . Detta är knappast realistiskt i praktiken, så vi får nog vara nöjda med fission och fusion!

Den ultimata energikällan vore naturligtvis antimateria. Antimateria finns emellertid inte tillgängligt utan måste tillverkas, se fråga 16650 .
/Peter E

Nyckelord: energikällor [26]; kärnenergi [19];

*

Universum-Solen-Planeterna [18896]

Fråga:
Hej! Stjärnor anges tillhöra population I eller population II, där solen tillhör den förra, yngre gruppen. En ännu äldre grupp skulle vara en population III, men den skall inte ha observerats. Beror det på att så gamla stjärnor har svalnat och inte strålar längre - de syns helt enkelt inte - men kan finnas ändå. Eller har de samtliga exploderat och ingår i övriga kvarvarande kroppar i universum?
/Thomas Å, Knivsta

Svar:
I dag använder man inte ofta klassificeringen i populationer utan något som kallas metallicitet, se Metallicity och länk 1.

Metallicitet avser inom astronomin ett objekts halt av "metall", grundämne, som inte är väte eller helium. Distinktionen är viktig eftersom väte, helium plus spår av litium antas vara de enda grundämnen som kan förekomma i universum utan föregående fusionsprocess i någon stjärna. Detta betyder att förekomst av metaller indikerar att stjärnor tidigare bildats och dött i en galax eller nebulosa. Metallicitet används ofta som ett mått på en stjärnas ålder: gamla stjärnor har låg metalliciet unga har hög.

De första stjärnorna (population III) var antagligen mycket massiva och därmed kortlivade (fråga 15342 ). Detta är antagligen skälet till att de ännu inte observerats: de måste befinna sig på mycket stora avstånd, vilket gör dem svåra att observera.

Se även länk 2.
/Peter E

Nyckelord: stjärnors utveckling [15];

1 http://burro.astr.cwru.edu/Academics/Astr222/Galaxy/Structure/metals.html
2 http://physics.stackexchange.com/questions/26271/how-can-a-population-iii-star-be-so-massive

*

Materiens innersta-Atomer-Kärnor [18929]

Fråga:
När en fusions/fissions bomb detonerar gör den ju sig beroende av sin omgivning. Ju mer massa som omringar den desto "bättre" effekt. Så vitt jag vet består inte rymden av mer än plasma gas och mörk materia/energi.

Detta bör då rimligtvis innebära att en vätebomb som detonerar i rymden får "sämre" effekt än på land. Men hur mycket förändras tryckvågen i rymden kontra land. Om vi utgår ifrån att en vätebomb detonerars bestående av 1kiloton sprängmedel(ca.140m tryckvåg på land)?
/Hampus O, Göteborg

Svar:
Det är mycket komplicerat att beräkna effekten av ett kärnvapen - man använder mycket sofistikerade simuleringsprogram för detta. Det är korrekt att man omger det klyvbara materialet med en s.k. "tamper". Detta är ett lager av ett tungt ämne som "håller ihop" det klyvbara materialet och reflekterar in neutroner. För ökad effekt använder man ofta utarmat uran (238U) som tamper eftersom det är klyvbart med snabba neutroner.

Ett kärnvapen i rymden har ganska begränsad verkan (se fråga 13679 ), utom för närområdet och, om explosionen sker nära jorden, kan det producera en EMP, se fråga 13095

Se vidare Neutron_reflector#Nuclear_weapons .
/Peter E

Nyckelord: kärnvapen [16];

*

Universum-Solen-Planeterna [18953]

Fråga:
Hej. Jag undervisar om universum, men har inte hittat svar på en viktig fråga. När en pulsar bildats så sänder den ut strålning vid polerna, vilket uppfattas som att den blinkar. Vad händer med den sen? Jag antar att strålningen är ett resultat av fusionsprocesser i stjärnan. Vad händer när kärnbränslet är slut? Vita dvärgar slocknar ju och blir svarta dvärgar, men vad händer med neutronstjärnor och svarta hål efter lång tid? Slocknar de också eller finns de kvar för evigt?

Mvh Bengt W
/Bengt W, Vallaskolan, Sollefteå

Svar:
Bengt! Nej, en neutronstjärna kan inte ha några fusionsprocesser. Den energikälla som finns hos en ensam pulsar är den snabba rotationen, se nedanstående citat fråga 13611 och Neutron_star#Rotation . Rotationen minskar alltså hela tiden, och slutresultatet blir ett svart kompakt objekt.

Om pulsaren är del av ett dubbelstjärnesystem kan material som dras in till neutronstjärnan snabba upp rotationen. Se fråga 15305 .

Från länk 1:
An example is the Crab pulsar, which is slowing its spin at a rate of 38 nanoseconds per day, releasing enough energy to power the Crab nebula.

Det termiska utvecklingen av en neutronstjärna beskrivs i länk 2.
/Peter E

1 http://astronomy.swin.edu.au/cosmos/N/Neutron+Star
2 http://www.astro.umd.edu/~miller/nstar.html#thermal

*

Materiens innersta-Atomer-Kärnor [19155]

Fråga:
Jag har några frågor om plasma av väte.

1. Vid vilken temperatur börjar plasma av väte uppträda.

2. Skulle 2 atomkärnor av väte, deuterium eller två andra mer lättreagearade ämnen, kunna fusionera, alltså slås ihop som i en fusion, bara genom att kollidera med varandra i hög fart?

3. Vilken hastighet har partiklar som fusionerar med varandra i en reaktor?


/Sören C, Västerås

Svar:
1. Bindningsenergin för väte är 13.6 eV vilket motsvarar en temperatur på c:a 100,000 K (EK=3kT/2). I själva verket räcker det pga Maxwell-Boltzmann fördelningen (Maxwell–Boltzmann_distribution ) med ett par tusen K för att väte skall vara till en del joniserat.

2,3. Ja, om bara energin är tillräckligt stor, c:a 10 keV eller 100 miljoner K.

Se vidare Tokamak .
/Peter E

*

Materiens innersta-Atomer-Kärnor [19345]

Fråga:
Varifrån kommer bindningsenergin som frigörs vid fusion?
/Veckans fråga

Ursprunglig fråga:
Hej! Jag har en fråga om bindningsenergin i atomkärnor. Vid fusion frigörs energi då neukloner får nya kopplingar av typen stark växelverkan. Kanske lite förenklat, men det här är väl i huvudsak rätt?

Vad jag har lite svårt att förstå är var den här energin kommer ifrån. Energi kan ju inte "bildas". När kärnorna slås ihop ökar bindningsenergin, men på något vis frigörs då energi. Hur kan energi "bli över" när det egentligen bildas nya kopplingar?

Jag har läst något om potentiell energi som finns "lagrad" i olika objekt och som sedan frigörs och bildar rörelseenergi då objekten växelverkar. Om jag förstår det rätt är det den här energin som kan frigöras i fusion. Dock så har väl den starka växelverkan begränsad räckvidd? Bär atomkärnorna ändå omkring på potentiell energi för ifall de hade kommit tillräckligt nära för att börja växelverka? Borde inte allting i universum då bära omkring på potentiell energi för ifall det skulle börja växelverka med något annat? Borde inte det finnas otroligt mycket energi lagrat som potentiell energi, energi lagrad för all annan materia i universum? Men så kan det väl inte vara?
/Axel H, Tunaskolan, Lund

Svar:
Hej Axel! Det första är helt korrekt: energi frigörs genom att nukleonerna binds till varandra. Den frigjorda energin kommer från en negativ potentiell energi i sluttillståndet. Låt oss anta vi har en samling fria nukleoner i vila. Energin är E=mc2, där m är massan. Om vi sätter samman nukleonerna till en kärna frigörs bindningsenergin U. Denna bindningsenergi kan användas t.ex. för att producera värme och elektricitet. Totala energin från början var ju E. Energin i sluttillståndet är då (E-U)+U=E (uttrycket i parentesen är kärnans totala energi). Vi ser att totala energin bevaras som den skall. Vi ser också att den sammansatta kärnan har mindre massa, M=(E-U)/c2, än de ingående nukleonerna, m=E/c2. Det är denna skillnad i massa som gör att vi kan få ut energi från vissa kärnreaktioner.

Hur stor bindningsenergin är beror på antalet nukleoner och på egenskaperna hos den starka kärnkraften. Om vi sätter samman t.ex. en järnkärna från neutroner och protoner blir bindningsenergin per nukleon c:a 9 MeV, se figuren i fråga 1433 . En nukleon har en viloenergi på ungefär 1 GeV, så den relativa energivinsten blir 9/1000=0.9%.

Se vidare fråga 18978 , 17569 och 13242 för tillämpningar med gravitationskraften och universum.
/Peter E

Nyckelord: potential/potentiell energi [30]; bindningsenergi [23];

*

Universum-Solen-Planeterna [19571]

Fråga:
Sker det någon utveckling i rymdteknologin när det gäller bemannade rymdresor till Mars?
/Veckans fråga

Ursprunglig fråga:
Det verkar som det inte händer så mycket inom rymdteknologin,när det gäller bemannade rymdresor till Mars eller längre ut,farten på skeppet behöver väl var hög så man kan ta sej långt så snabbt som möjligt.Finns det någon teknik inom närmaste framtid,som gör det möjligt att ta oss fram relativt snabbt i rymden?
/Jörgen B, Kista

Svar:
Nej, det finns ingen revolutionerande ny teknik vad gäller bemannade rymdfärder. För obemannade rymdfärder utvecklas emellertid små, avancerade och relativt billiga prober som kan sändas till olika objekt i solsystemet.

NASA har varit marginellt involverat i kall fusion (kall fusion ). Trots att detta område är ganska kontroversiellt spekulerade man mycket om att använda kall fusion som framdrivning. Tyvärr stannar det ganska säkert vid spekulationer - kall fusion finns inte!

Sedan finns det projekt att sända några människor på en enkel resa till Mars. Förvånansvärt nog finns det många anmälda, vi får se hur många som återstår om det någonsin kommer till kritan. Jag tror att det bara är ett projekt att lura pengar av godtrogna.

Det finns två problem som gör marsfärder svåra och därmed dyra. Det tar lång tid att färdas till Mars (se fråga 17360 ) och passagerarna kan komma att utsättas för höga stråldoser, speciellt från solaktiviteten.

Tekniskt skulle vi kunna skicka en bemannad farkost till Mars tur och retur i dag, men till enorma kostnader. Vinsten jämfört med utforskning av Mars med obemannade prober är marginell. Möjligen kan bemannade färder bli aktuella på 2030-talet.

Vad gäller färder längre bort i solsystemet planeras endast obemannade prober. Mest spännande är New Horizons (se New_Horizons ) till dvärgplaneten Pluto och JUICE (se Jupiter_Icy_Moon_Explorer ) till Jupiters ismånar.

Se vidare om olika aspekter av marsfärder i några befintliga svar: Mars . Se även en artikel om marsfärder i pappersversionen av Populär astronomi, länk 1.



/Peter E

Nyckelord: Mars [12]; rymdfärder [23];

1 http://www.popast.nu/arkiv/nummer-4-december-2014

*

Universum-Solen-Planeterna [19629]

Fråga:
Kan människor skapa en stjärna?
/Eek d, Vibyskolan, Sollentuna

Svar:
Nej, det kan vi inte. En stjärna måste ha en massa på minst 10% av solens massa. Så mycket helium/väte kan vi inte hantera.

Vi kan däremot skapa de fusionsreaktioner som ger stjärnorna energi (fråga 10658 ) genom kontrollerad fusion med tokamak eller okontrollerat i vätebomber.
/Peter E

*

Energi [19726]

Fråga:
Kan vi vinna något på att omvandla/utvinna energi på månen?
/Veckans fråga

Ursprunglig fråga:
Energitransport från månen.

Kan vi vinna något på att omvandla/utvinna energi på månen? Om vi gör det, hur skulle vi kunna överföra denna energi till jorden?
/Yusein T, st.erisk, kista

Svar:
Ett förslag är att ta upp He-3 (som kommer med solvinden) och transportera detta till jorden för användning i fusionsreaktorer, se länk 1. Problemet är dels att man inte vet om det finns något He-3 och man har ingen fungerande reaktordesign, se länk 1 och 2.

Lite mer realistiskt är att elektrolysera vatten (från befintlig is) med sol-el och transportera vätgas till jorden. Detta lär dock knappast vara lönsamt eftersom det finns massor med vatten och solsken på jorden.

I övrigt känner man inte till några energikällor på månen. Fossila bränslen saknas naturligtvis eftersom det aldrig funnits något liv.

Det finns för närvarande inga bra skäl för människan att etablera sig på månen. Detta är anledningen till att inga bemannade månfärder har utförts sedan 1972 (Apollo 16).
/Peter E

Nyckelord: fusion [17]; energikällor [26]; månfärder [7];

1 http://www.esa.int/Our_Activities/Preparing_for_the_Future/Space_for_Earth/Energy/Helium-3_mining_on_the_lunar_surface
2 http://iec.neep.wisc.edu/index.php

*

Universum-Solen-Planeterna [19902]

Fråga:
Hur skulle Universum vara ifall gravitationen var en svagare kraft/materia påverkade rumtiden mindre? Skulle avancerat liv då kunnat utvecklas på en planet mycket större än Jorden? Skulle det kunna finnas en mycket större diversitet av liv på en sådan hypotetisk planet? Hur skulle horisonten se ut? Skulle tryck och graviation bete sig likadant ifall planeten var mer massiv och hade en större atmosfär? Skulle planeten ha ett kraftigare magnetfält? Skulle där vara en större vulkanisk aktivitet? Skulle man kunna se längre?

Skulle bli mycket glad ifall jag fick detaljerade svar på dessa frågor och hur ett Universum med en svagare gravitation skulle se ut. Jag är också intresserad av hur snabbt stjärnors fusion skulle gå och hur långlivade de skulle vara i ett sådant hypotetiskt Universum.
/Philip A, Nya Malmö Latin, Malmö

Svar:
Philip! Det var en massa frågor, varav flera är svåra eller omöjliga att svara på!

Generellt är värden på naturkonstanter av stor betydelse för universums utveckling. Även ganska små avvikelser från de värden vi kan mäta upp gör galaxer, stjärnor, planeter och liv omöjliga. Stjärnor behövs ju inte bara för att ge planeter lagom temperatur för liv utan även för att bygga upp ämnen som är tyngre än väte och helium (vilka ju bildades vid big bang).

Det finns alltså ett antal naturkonstanter som måste ha det aktuella värdet för att planeter och liv skall uppstå. Är detta en tillfällighet? Nej, naturligtvis inte. Med avvikande värden skulle vi inte existera, och alltså inte kunna fundera på varför konstanterna är vad de är (Physical_constant#Anthropic_principle ).

Se vidare Fysikalisk_konstant , länk 1 och 2 nedan.
/Peter E

Nyckelord: kosmologi [33]; naturkonstant [7];

1 http://www.focus.org.uk/gravity.php
2 https://sciencebits.wordpress.com/2008/08/26/what-if-gravity-was-weaker-or-stronger/

*

Materiens innersta-Atomer-Kärnor [19986]

Fråga:
Enorm energikälla-iron man?

Iron Man, vad kan han tänkas använda för energikälla och isånnafall hur fungerar den? Det behöver inte vara någon sorts verklighets trogen energikälla, men en som ändå (eventuellt)skulle kunna vara möjlig men som man bara inte har tillgång till idag. Om jag skulle vara Iron Man, vilken form av energikälla är mest lämplig för dräkten? Fri fantasi!
/Ludvig J, Kungsfågeln, Luleå

Svar:
Jag kan tänka mig att kall fusion (se fråga 2409 ) är tillräckligt otroligt för att passa i en amerikansk actionfilm, Iron_Man_(2008_film) . Antingen Rossis E-Cat (17662 ) eller Mills Blacklight Power (14237 ) borde platsa bra!

Kall fusion har förekommit i filmer: Chain_Reaction_(film)#Scientific_accuracy och The_Saint_(film)#Plot .
/Peter E

Nyckelord: kall fusion [8];

*

Materiens innersta-Atomer-Kärnor [20113]

Fråga:
Dimkammare
/Veckans fråga

Ursprunglig fråga:
Hej, till mitt gymnasiearbete har jag konstruerat en dimkammare men jag har lite svårigheter med att identifiera all strålning. Skulle ni kunna förklara hur myoner, alfa partiklar, beta partiklar o.s.v. ser ut i en dimkammare?
/Emelie J

Svar:
De vanligaste spåren är av a-partiklar (korta, raka, tjocka) och elektroner (långa, krokiga, tunna).

Nedanstående bild från Cloud_chamber visar flera olika spår av partiklar, se bildtexten.

I länk 1 finns en intressant bild som innehåller dubbla V-formade a-spår. Det är 220Rn (från sönderfallskedjan 232Th, se fråga 13744 ) som sönderfaller till 216Po. 216Po är mycket kortlivat (0.14 s), så dotterkärnan sönderfaller innan den hunnit flytta på sig. Om man tittar noga kan man se att det är en liten fördröjning mellan spåren.

This rare picture show the four types of charged particles that we can detect in a cloud chamber : alpha, proton, electron and muon (probably). Picture taken at the Pic duMidi at 2877 m in a Phywe PJ45 diffusion cloud chamber. Size of the interaction surface if 45x45 cm.


/Peter E

Nyckelord: radioaktivitet, sönderfallskedja [7]; dimkammare [2];

1 https://en.wikipedia.org/wiki/Cloud_chamber#/media/File:Radon220_decay_in_a_cloud_chamber.jpg

*

[20288]

Fråga:
Enligt de omfattande beräkningsmodeller jag genomsökt borde Jordens-, solens absoluta centrum och även centrum i ett svart hål uppvisa antigravitation. Vilket innebär att fusionsprocessen i solen sker på ytan av ett stort supertungt plasmaklot. Jag tycker mig kunna påvisa detta i de beräkningsmodeller jag utfört? Kommentera gärna detta.
/P S, Linköping

Svaret kommer snart...

*

Universum-Solen-Planeterna [20335]

Fråga:
Supermassiva svarta hål
/Veckans fråga

Ursprunglig fråga:
I centrum av galaxer, t ex Vintergatan, finns det ett svart hål (sägs det). Har det svarta hålet varit med från galaxens "födelse" eller har det tillkommit senare?
/Thomas Å, Knivsta

Svar:
Supermassiva svarta hål är förmodade svarta hål med massa motsvarande miljoner eller miljarder gånger solens massa. De flesta, kanske alla, galaxer tros ha ett supermassivt svart hål i sitt centrum. Det gäller även vår galax Vintergatan, där objektet Sagittarius A* i centrum med största sannolikhet är ett supermassivt svart hål. (Supermassivt_svart_hål )

Det finns ännu ingen etablerad detaljerad teori för hur galaxer bildas. Det är klart att galaxer uppstår ur kollapsande gasmoln och att stjärnorna uppstår när molnet fragmenteras och fragmenten kollapsar. På något sätt hjälper nog den mörka materien till i processen.

Om alla galaxer (åtminstone stora galaxer) innehåller ett supermassivt svart hål, är det nog naturligt att anta att detta uppkommer som en del av galaxbildningen. Kan t.ex. centrum av en blivande galax tänkas nå tillräcklig densitet utan att fusionsprocesser startas? Fusionprocesserna värmer ju upp gasen och om massan är stor dominerar strålningstrycket över gravitationen och den nybildade stjärnan slits itu. Men om massan är jättestor, eventuellt med hjälp av mörk materia (som inte fusionerar), kanske ett svart hål kan bildas.

I ett större antal aktiva galaxer och kvasarer (se fråga 13916 ), visar sig det supermassiva hålet genom kärnans aktivitet, nämligen genom den enorma mängd strålning som kommer från centrum, vilken tros härstamma från gas som cirkulerar in i hålet. Man antar att de flesta ljusstarka galaxer har ett supermassivt svart hål, men att de flesta är i "inaktivt" läge där de inte drar till sig speciellt mycket materia.

Det uppskattas att supermassiva svarta hål skapas om tillräckligt många stjärnor befinner sig på ett tillräckligt litet område i rymden eller tillräckligt många sugs in i ett ursprungligt svart hål, alternativt om flera svarta hål slås samman. De nödvändiga förutsättningarna för detta tros finnas allmänt i centrum av större galaxer. Teoretiska studier av kollapser av tunga stjärnor visar att extremt tunga stjärnor (flera hundra solmassor) kan kollapsa i sin helhet till svarta hål, vilket kunnat vara frön till supermassiva svarta hål. Så extremt tunga stjärnor tros bara kunnat bildas i frånvaro av grundämnen tyngre än helium, något som bara gällde den första tiden efter Big Bang.

Det finns exempel på galaxer som har mer än ett supermassivt svart hål. Dessa har troligtvis uppkommit genom sammanslagning av två galaxer. Galaxer är relativt stora jämfört med avståndet mellan dem, så kollisioner är ganska vanliga.

Se även fråga 6228 och Supermassive_black_hole .
/Peter E

Nyckelord: svart hål [51]; galax [28]; kvasar [4]; Vintergatan [6];

1 https://science.nasa.gov/astrophysics/focus-areas/black-holes/
2 http://www.spacetelescope.org/science/black_holes/

*

[20340]

Fråga:
Det är beklagligt att man ännu inte lyckats åstadkomma en fusionsreaktor här på jorden liknande Solen. Väte kärnor slås ihop och bildar energin. Nu har man övergett den teorin eftersom man TROR att det enorma tryck som behövs inte kan framställas på jorden. Istället har man gått över till att använda tungt väte, deuterium och nått annat kommer inte ihåg vad och försöka åstadkomma fusion vid högre värme och lägre tryck, detta skapar tyvärr även en radioaktiv biprodukt. Försöken har dessvärre inte kommit så vidare långt heller. Jag menar att man visst kan skapa det tryck som behövs, med betoning på TROR (vad man visserligen skall göra i kyrkan) att det är möjligt. Väte plasman har en tendens att uppföra sig som den vill i höga temperaturer Svårt att hålla på plats med magnetfält, vi kan se samma sak i Solen magnetstormar. Vad kan åstadkomma högt tryck på jorden? jo cykloner, tromber, hurricanes. En American tog ett patent på en konstgjord cyklon, han sålde konstigt nog till ett Svenskt företag i Skellefteå som skulle ha tekniken att torka räkskal etc Man skall tydligen även lyckats separera salt ur havsvatten. Företaget heter Airgrind och har förmodligen samlat en del kunskap om detta, visserligen inte i denna skala som ni behöver men ändå. Jag tror att ett batteri av högeffektiva konstgjorda cykloner kan skapa det tryck ni eftersträvar samt + även magnetfält, problemet lär bli att hålla processen i schack, möjligen oscillerande tryck så att hela skiten inte smäller av. Hoppas ni fått eventuella ny infallsvinklar eller lösning på problemet, vore kul om Sverige blev först att lösa världens energi behov.
/Evald T, KTS, Katrineholm

Svaret kommer snart...

*

Ljud-Ljus-Vågor [20364]

Fråga:
Har förstått att en lampa blinkar på 50hz? Vad har Hz för samband med ögat? Vad är gränsen för hur mycket Hz ögat kan uppfatta. Gör gärna en fördjupning i det området, snabb kurs på induktion och transformation också tack:)
/Kevin M, Sofiaskolan, Södermalm

Svar:
Nej, en glödlampa blinkar med frekvensen 100 Hz eftersom ljusintensiteten är proportionell mot effekten R*I2. Detektorerna i ögat (stavar och tappar) svarar inte oändligt snabbt på varierande ljusintensitet, men frekvenser över 70 Hz uppfattas inte som varierande av de flesta människor. En glödlampa blinkar dessutom med liten amplitud eftersom den inte hinner svalna så mycket när växelspänningen är låg. Se fråga 20105 och 12034 .

Se även Flicker_(screen) och Flicker_fusion_threshold .

Se fråga 15153 för induktion och fråga 12295 för transformator.
/Peter E

Nyckelord: ögat [18]; glödlampa [23];

*

Blandat [20540]

Fråga:
Frågor kring Lgr 11

Jag prövar att förstå Lgr 11 för år 4-6 (jag är ny lärare i en år 4), men har svårt att förstå vissa bitar. Så jag har lite frågor.

Den första är:

1. När jag ska lära eleverna hur man resonerar har jag följande mall som stöd. Hur tänker ni den skulle fungera för en år 4?

- Påstående
- Förklaring

Jag har valt ordet ”påstående” eftersom jag inte hittar något bättre. Det är ett nytt ord för dom, det kanske blir för mycket när resonera också är ett nytt ord för dom. Öppen för annat.

Förklaring tycker jag är rätt bra då dom redan känner till det ordet. Sen vet jag att man kan bygga vidare med

2. Vad är fysikaliska samband, i t ex elektriska kretsar? Är det naturlagen som syftes till? Kunna du ge ett exempel i så fall?
/Jeppe S, Ekarängskolan, Borås

Svar:
Fysik är en del av naturvetenskap. Låt oss först se vad naturvetenskap är.

Naturvetenskap är läran om den fysiska världen. Naturvetenskapen använder vetenskapliga metoder som i hög grad beror på empiriska mätningar av uppställda hypoteser (vetenskaplig metod, se fråga 14237 ). Naturvetenskapen försöker göra teoretiska modeller som så enkelt som möjligt kan förklara det som kan observeras, mätas och testas (se Ockhams rakkniv i fråga 3810 ).

1 Mja, jag är tveksam till "påstående". Det låter som något som Donald Trump hittar på på nätterna ! Jag tycker att man från början skall göra klart att fysik är en empirisk vetenskap. Jag skulle föredra "observation". Det täcker både observationer av företeelser vi inte kan påverka (t.ex. supernovautbrott) och resultatet av ett genomfört experiment (t.ex. samband mellan spänning och ström).

"Förklaring" är OK om man bara inte övertolkar det som svar på frågan "varför?". Fysiken är som den är.

2 Våra förklaringar utgörs ofta matematiska samband mellan olika storheter. För elektriska kretsar t.ex. Ohms lag och Kirchhoffs lag, se fråga 15837 .

Tro, hopp och fysik

Man skall även ha klart för sig att fysik är en mycket mogen vetenskap: vår förståelse av fysikaliska fenomen är mycket god vad gäller grundläggande fenomen, t.ex. atomfysik, kärnfysik och kvantmekanik. Detta betyder att bevisbördan är mycket tung för den som hävdar sig ha upptäckt något som strider mot etablerad kunskap. Nyheter om upptäckten av nya sorters kärnreaktioner är ett exempel som förekommit flera gånger i över trettio år. Anledningen till att sådana "fake news" (för att använda en aktuell term) får så stort genomslag är människors tendens att tro på det de vill tro på (vi kan lösa alla problem med global uppvärmning med en liten apparat man kan tillverka i garaget) i stället för att kritiskt granska de bevis som finns. Öppen publicering, reproducerbarhet och kritik är grunderna för god vetenskap.

Se vidare fråga 2409 (kall fusion) och 14237 (vetenskaplig metod).
/Peter E

Nyckelord: kursplan [3]; fysik, förståelse av [17];

1 https://www.skolverket.se/laroplaner-amnen-och-kurser/grundskoleutbildning/grundskola/fysik

*

[20796]

Fråga:
6.Hur mycket energifrigörs vid fusion av 1 g väte?

7.Hur många villaårsförbrukningar motsvarar denna energimängd om vi utgår från att varje villa använder 20000 kWh/år.

8.Beräkna den energi som frigörs vid fission av U-235 enligt följande process: 01415692360235923nBaKrnUoch beräkna därefter hur många villors årsförbrukning det motsvarar om man klyver 1 g U-235.
/dalal j, kattegatt, halmstad

Svaret kommer snart...

*

Energi [20816]

Fråga:
Vilken energikälla är bäst för framtiden och varför?
/Veckans fråga

Ursprunglig fråga:
Vilken energikälla är bäst för framtiden och varför?
/Alex B, Sandagymnasiet, Huskvarna

Svar:
Det beror på vad man skall ha energin till. Praktiska och ekonomiska begränsningar ger olika lösningar för t.ex. transporter och elproduktion.

På sikt kommer säkert (definitionsvis!) förnybara energikällor att dominera. Nästan alla förnybara energikällor (1782 ) använder solen som primär källa.

De viktigaste kommer antagligen bli de källor som utvecklas snabbt för närvarande - vindkraft (19115 ) och solceller (1129 ). Vattenkraft (19360 , 12463 ) kommer säkert att finnas kvar. Kärnkraft kan tänkas komma i form av fusion (14847 ).

En viktig komponent är energisparande. Belysning har under de senaste åren utvecklats från några få procents verkningsgrad till nästan 100% ljusutbyte med RGB lysdioder (16165 ).

Se även Energy_development , solenergi och länk 1/2 nedan.
/Peter E

Nyckelord: energikällor [26];

1 https://science.howstuffworks.com/environmental/energy/energy-source-future.htm
2 http://www.visualcapitalist.com/alternative-energy-sources-future/

*

Elektricitet-Magnetism [20901]

Fråga:
Är det bara järn som är magnetiskt? Eller är alla ämnen mer eller mindre magnetiska? Tycker mig minnas att vid fusion så hålls väteatomerna fast av magnetfält?
/Mats P

Svar:
Starkt magnetiska är endast järn, kobolt och nickel - ferromagnetiska ämnen, se fråga 12402 .

Magnetism är intimt kopplad till elektriska fält. Bland annat skapas magnetfält av en elektrisk laddning som rör sig. Detta fält kan växelverka med ett yttre fält. I en fusionsplasma är atomerna joniserade av den höga temperaturen. De laddade jonerna kan då hållas inneslutna.
/Peter E

Nyckelord: ferromagnetism [9];

*

Materiens innersta-Atomer-Kärnor [20998]

Fråga:
För att sätta igång reaktionen i en fusionsbomb måste det först sprängas en fissionsbomb, så att trycket och temperaturen blir tillräckligt hög. Varför gör man då inte likadant för att skapa en reaktion i en fusionsreaktor? Först sker en kärnklyvning på samma sätt som i dagens kärnkraftvärk som sedan skapar de rätta förutsättningarna för att låta en fusion ske. På så sätt skulle det krävas mindre andel skadliga ämnen, för det krävs bara det lilla som skapar de rätta förutsättningarna för en fusion.
/Stella S, Vasaskolan, Göteborg

Svar:
Tyvärr är det inte så enkelt.

Man skulle kunna tänka sig att fånga in neutroner från fissionsreaktioner och använda det lilla överskottet av neutroner för att producera energi. Utbytet av energi är dock alldeles för litet eftersom sannolikheten att neutronen reagerar (tvärsnittet för reaktionen) är för liten.

I en fusionsbomb är det inte neutronerna som skapar fusionsreaktionerna utan högt tryck och hög temperatur. Denna våldsamma explosion går emellertid inte att hantera för att utvinna energi för nyttiga ändamål.

Se fråga 14847 för mer om fission/fusion och fråga 1351 för ett alternativt system för fissionsreaktorer.
/Peter E

Nyckelord: fusion [17];

*

Energi [21048]

Fråga:
Hej! Om man söker på följande mening i youtube: make electric free energy using magnet with spark plug, så visas ett experiment med två tändstift som vardera lindas med en ledning. Tändstiften limmas fast på varsin rund magnet. Sen kopplas ledningarna till en lampa som då lyser konstant. Jag undrar hur lampan kan lysa. Vore det induktion skulle man ju behöva röra på magneterna, men allt ligger stilla. Något batteri ser jag inte heller.
/Per Å, Kalmar

Svar:
Detta är ett exempel (av många) på "gratis energi" (free energy):

Du gör helt rätt i av tvivla på sanningshalten. Experimentet strider ju mot det allra heligaste i fysiken: lagen om energins bevarande (energiprincipen eller termodynamikens första huvudsats), se fråga 15733 .

För den som inte litar på fysikens lagar kan vi ta till kommersiella lagar: om apparaten fungerar som det påståtts så hade all belysning använt metoden!

Utan att undersöka uppställningen är det omöjligt att säga hur fusket går till, men av man ser är det sannolikt at man har en extra variabel magnet gömd under bordet. Spolarna plockar upp det varierande magnetfältet och transformerar det till högspänning som får lampan att lysa.

Engelska Wikipedia säger följande om gratis energi ( Free_energy ):

Pseudoscience:
* Free energy device, a hypothetical perpetual motion device that creates energy, thereby contradicting the laws of thermodynamics
* Free energy suppression conspiracy theory, in which advanced energy technologies are suppressed by governments or special interest groups

Frågor under nyckelordet Kall fusion innehåller diskussion om andra energibluffar som lurat mycket pengar av naiva investerare.

Se även fråga 13608 .
/Peter E

Nyckelord: pseudovetenskap [11];

*

Materiens innersta-Atomer-Kärnor [21146]

Fråga:
Hej! Jag håller på med ett gymnasiearbete som handlar om fusion och försöker kolla lite på tröskelenergi men har inte hittat något sätt att räkna ut den på, inte heller någon bra härledning till det.

Jag har också förstått att det finns något som heter lawson kriteriet men har inte förstått om det har med detta att göra eller vad det ens är.
/Linus S, Bäckängsgymnasiet

Svar:
Tröskelenergi är ett begrepp som används i samband med kärnreaktioner med negativt Q-värde (reaktioner som kostar energi). För att en sådan reaktion skall kunna ske måste extra energi tillföras eftersom en del av energin går åt för att bevara rörelsemängden i sluttillståndet, se Threshold_energy . Jag kan inte se att tröskelenergi är relevant för fusion eftersom det då rör sig om reaktioner som frigör energi - reaktioner med positivt Q-värde, t.ex. d+3H.

Lawsonkriteriet (se Lawson_criterion och länk 1,2) är ett mått (en s.k. "figure of merit", se Figure_of_merit ) på hur effektivt ett fusionssystem är. Det definieras som trippelprodukten

n t T

där n är elektrondensiteten, t är inneslutningstiden och T är temperaturen hos plasmat. Lawsons ursprungliga definition innehöll inte T.

Se även fusion .
/Peter E

Nyckelord: fusion [17]; rörelsemängd [15];

1 http://hyperphysics.phy-astr.gsu.edu/hbase/NucEne/lawson.html
2 https://www.iter.org/newsline/261/1527

*

Materiens innersta-Atomer-Kärnor [21149]

Fråga:
Hej! Jag håller på med ett gymnasiearbete som handlar om fusion och försöker kolla lite på tröskelenergi men har inte hittat något sätt att räkna ut den på, inte heller någon bra härledning till det. Jag har också förstått att det finns något som heter lawson kriteriet men har inte förstått om det har med detta att göra eller vad det ens är.
/Linus S, Bäckängsgymnasiet

Svar:
Jag har redan svarat på din fråga, se fråga 21146
/Peter E

*

Sök efter    

Skriv de ord du vill söka på i sökfältet ovan och klicka på sökknappen. Uteslut ord genom att sätta - (minus) före ordet. Ordgrupper definieras med hjälp av "...". Sökningar är oberoende av stora och små bokstäver.

Exempel:

helium "kalle anka"
Sök på 'helium' och ordgruppen 'kalle anka'
orgelpipa
Sök på 'orgelpipa'
orgel -gitarr
Sök på 'orgel' men inte 'gitarr'

 


sök | söktips | Veckans fråga | alla 'Veckans fråga' | ämnen | dokumentation | ställ en fråga
till diskussionsfora

 

Creative Commons License

Denna sida från NRCF är licensierad under Creative Commons:
Erkännande-Ickekommersiell-Inga bearbetningar
.